Rules Flashcards

1
Q

Missing parts of the description or missing drawings Rule 56 (1)

A

If the examination under Article 90, paragraph 1, reveals that parts of the description, or drawings referred to in the description or in the claims, appear to be missing, the European Patent Office shall invite the applicant to file the missing parts within two months. The applicant may not invoke the omission of such a communication.

How well did you know this?
1
Not at all
2
3
4
5
Perfectly
2
Q

Missing parts of the description or missing drawings Rule 56 (2)

A

If missing parts of the description or missing drawings are filed later than the date of filing, but within two months of the date of filing or, if a communication is issued under paragraph 1, within two months of that communication, the application shall be re-dated to the date on which the missing parts of the description or missing drawings were filed. The European Patent Office shall inform the applicant accordingly.

How well did you know this?
1
Not at all
2
3
4
5
Perfectly
3
Q

Missing parts of the description or missing drawings Rule 56 (3)(a)

A

If the missing parts of the description or missing drawings are filed within the period under paragraph 2, and the application claims priority of an earlier application, the date of filing shall, provided that the missing parts of the description or the missing drawings are completely contained in the earlier application, remain the date on which the requirements laid down in Rule 40, paragraph 1, were fulfilled, where the applicant so requests and files, within the period under paragraph 2:

a copy of the earlier application, unless such copy is available to the European Patent Office under Rule 53, paragraph 2;

How well did you know this?
1
Not at all
2
3
4
5
Perfectly
4
Q

Missing parts of the description or missing drawings Rule 56 (3)(b)

A

If the missing parts of the description or missing drawings are filed within the period under paragraph 2, and the application claims priority of an earlier application, the date of filing shall, provided that the missing parts of the description or the missing drawings are completely contained in the earlier application, remain the date on which the requirements laid down in Rule 40, paragraph 1, were fulfilled, where the applicant so requests and files, within the period under paragraph 2:

where the earlier application is not in an official language of the European Patent Office, a translation thereof in one of these languages, unless such copy is available to the European Patent Office under Rule 53, paragraph 3;

How well did you know this?
1
Not at all
2
3
4
5
Perfectly
5
Q
Missing parts of the description or missing drawings
Rule 56 (3)(c)
A

If the missing parts of the description or missing drawings are filed within the period under paragraph 2, and the application claims priority of an earlier application, the date of filing shall, provided that the missing parts of the description or the missing drawings are completely contained in the earlier application, remain the date on which the requirements laid down in Rule 40, paragraph 1, were fulfilled, where the applicant so requests and files, within the period under paragraph 2:

an indication as to where the missing parts of the description or the missing drawings are completely contained in the earlier application and, where applicable, in the translation thereof.

How well did you know this?
1
Not at all
2
3
4
5
Perfectly
6
Q
Missing parts of the description or missing drawings 
Rule 56 (4)(a)
A

If the applicant:

fails to file the missing parts of the description or the missing drawings within the period under paragraph 1 or 2,

any references referred to in paragraph 1 shall be deemed to be deleted, and any filing of the missing parts of the description or missing drawings shall be deemed not to have been made. The European Patent Office shall inform the applicant accordingly.

How well did you know this?
1
Not at all
2
3
4
5
Perfectly
7
Q
Missing parts of the description or missing drawings 
Rule 56 (4)(b)
A

withdraws under paragraph 6 any missing part of the description or missing drawing filed under paragraph 2,

any references referred to in paragraph 1 shall be deemed to be deleted, and any filing of the missing parts of the description or missing drawings shall be deemed not to have been made. The European Patent Office shall inform the applicant accordingly.

How well did you know this?
1
Not at all
2
3
4
5
Perfectly
8
Q
Missing parts of the description or missing drawings
Rule 56 (5)
A

If the applicant fails to comply with the requirements referred to in paragraph 3(a) to (c) within the period under paragraph 2, the application shall be re-dated to the date on which the missing parts of the description or missing drawings were filed. The European Patent Office shall inform the applicant accordingly.

How well did you know this?
1
Not at all
2
3
4
5
Perfectly
9
Q

Missing parts of the description or missing drawings Rule 56 (6)

A

within one month of the notification referred to in paragraph 2 or 5, last sentence, the applicant may withdraw the missing parts of the description or the missing drawings filed, in which case the re-dating shall be deemed not to have been made. The European Patent Office shall inform the applicant accordingly.

How well did you know this?
1
Not at all
2
3
4
5
Perfectly
10
Q

Form of the drawings - Rule 46 (1)

A
On sheets containing drawings, the usable surface area shall not exceed 26.2 cm x 17 cm. The usable or used surface shall not be surrounded by frames. The minimum margins shall be as follows: 
top 2.5 
cm 
left side 2.5 
cm 
right side 1.5 
cm 
bottom 1 
cm
How well did you know this?
1
Not at all
2
3
4
5
Perfectly
11
Q

Form of the drawings - Rule 46 (2)

A

Drawings shall be executed as follows:
(a)
Drawings shall be executed without colourings in durable, black, sufficiently dense and dark, uniformly thick and well-defined lines and strokes.
(b)
Cross-sections shall be indicated by hatching which should not impede the clear reading of the reference signs and leading lines.
(c)
The scale of the drawings and their graphical execution shall be such that electronic or photographic reproduction with a linear reduction in size to two-thirds will allow all details to be distinguished without difficulty. If, exceptionally, the scale is given on a drawing, it shall be represented graphically.
(d)
All numbers, letters, and reference signs appearing on the drawings shall be simple and clear. Brackets, circles or inverted commas shall not be used in association with numbers and letters.
(e)
Generally, all lines in the drawings shall be drawn with the aid of drafting instruments.
(f)
Elements of the same figure shall be proportional to one another, unless a difference in proportion is indispensable for the clarity of the figure.
(g)
The height of the numbers and letters shall not be less than 0.32 cm. For the lettering of drawings, the Latin and, where customary, the Greek alphabets shall be used.
(h)
The same sheet of drawings may contain several figures. Where figures drawn on two or more sheets are intended to form a single figure, the figures on the several sheets shall be so arranged that the whole figure can be assembled without concealing any part of the partial figures. The different figures shall be arranged without wasting space, preferably in an upright position, clearly separated from one another. Where the figures are not arranged in an upright position, they shall be presented sideways with the top of the figures at the left side of the sheet. The different figures shall be numbered consecutively in Arabic numerals, independently of the numbering of the sheets.
(i)
Reference signs not mentioned in the description and claims shall not appear in the drawings, and vice versa. Reference signs to features shall be consistent throughout the application.
(j)
The drawings shall not contain text matter. Where indispensable to understand the drawings, a few short keywords, such as “water”, “steam”, “open”, “closed” or “section on AB”, may be included. Any such keywords shall be placed in such a way that, if required, they can be replaced by their translations without interfering with any lines of the drawings.

How well did you know this?
1
Not at all
2
3
4
5
Perfectly
12
Q

Form of the drawings - Rule 46 (3)

A

Flow sheets and diagrams shall be deemed to be drawings.

How well did you know this?
1
Not at all
2
3
4
5
Perfectly
13
Q

True or False

For final grant, a European patent application requires the submission of:

Claims.

A

True.

Claims are not required for the act of filing and a filing date can be accorded even without a single claim. However, the extent of protection of a European patent is determined by the claims (Article 69 EPC). Therefore at least one claim is a prerequisite for obtaining patent protection and a patent cannot be granted without a claim. Note that, even before grant, a European patent application must contain at least one claim (Article 78(c) EPC). The extent of provisional protection of a European patent application is determined by its claims as published (Article 69(2) EPC). Consequently, even though claims are not required immediately on filing, they are required soon afterwards, as stipulated in Article 78(c) EPC.

How well did you know this?
1
Not at all
2
3
4
5
Perfectly
14
Q

True or False

For final grant, a European patent application requires the submission of:

Drawings.

A

False.

Depending on the subject-matter, a drawing or drawings may or may not be required

How well did you know this?
1
Not at all
2
3
4
5
Perfectly
15
Q

True or False

For final grant, a European patent application requires the submission of:

An abstract.

A

True.

To cover the requirement to inform the public, an abstract is required and is asked for in the application. If not submitted upon filing, it will have to be furnished later (see Article 78(1)(e), Rule 57(d), Rule 58 and Article 85 EPC).

How well did you know this?
1
Not at all
2
3
4
5
Perfectly
16
Q

True or False

For final grant, a European patent application requires the submission of:

A translation of the complete description into all three EPO languages.

A

False.

see Rule 71(3) EPC

How well did you know this?
1
Not at all
2
3
4
5
Perfectly
17
Q

True or False

It is possible to file a European patent application:

A. By email.
B. By fax.
C. With the EPO in Berlin.
D. With the EPO in Vienna.

A

False, True, True, False

See Rule 35 EPC

How well did you know this?
1
Not at all
2
3
4
5
Perfectly
18
Q

Although not sufficient in itself, which of the following can contribute to the requirements for according a filing date?

A. The file number of a previous application.
B. A contact address for the applicant.
C. The names of a group of applicants.
D. A complete description of the invention.

A

All of them

See Rule 40 EPC

How well did you know this?
1
Not at all
2
3
4
5
Perfectly
19
Q

Date of Filing - Rule 40 (1)

A

The date of filing of a European patent application shall be the date on which the documents filed by the applicant contain:

(a) an indication that a European patent is sought;
(b) information identifying the applicant or allowing the applicant to be contacted; and
(c) a description or reference to a previously filed application.

How well did you know this?
1
Not at all
2
3
4
5
Perfectly
20
Q

Date of Filing - Rule 40 (2)

A

A reference to a previously filed application under paragraph 1(c) shall state the filing date and number of that application and the Office with which it was filed. Such reference shall indicate that it replaces the description and any drawings.

How well did you know this?
1
Not at all
2
3
4
5
Perfectly
21
Q

Date of Filing - Rule 40 (3)

A

Where the application contains a reference under paragraph 2, a certified copy of the previously filed application shall be filed within two months of filing the application. Where the previously filed application is not in an official language of the European Patent Office, a translation thereof in one of these languages shall be filed within the same period. Rule 53, paragraph 2, shall apply mutatis mutandis.

How well did you know this?
1
Not at all
2
3
4
5
Perfectly
22
Q

Which of the following will lead to an application not being treated as a European patent application?

A. There is no indication at all about the applicant after two months of receipt of the application documents by the EPO.
B. In the case of filing by reference, a certified copy of the previous application is submitted six weeks after notification that the certification is missing.
C. A translation of the application was required and was filed nine weeks after an invitation under Rule 55 EPC.
D. It is not clear from the filed documents that a European patent is sought.

A

True, False, False, True

How well did you know this?
1
Not at all
2
3
4
5
Perfectly
23
Q

Examination on filing - Rule 55 EPC

A

If the examination under Article 90, paragraph 1, reveals that the application fails to meet the requirements laid down in Rule 40, paragraph 1(a) or (c), paragraph 2 or paragraph 3, first sentence, the European Patent Office shall inform the applicant of any deficiencies and advise him that the application will not be dealt with as a European patent application unless such deficiencies are remedied within two months. If the applicant does this, he shall be informed of the date of filing accorded by the Office.

How well did you know this?
1
Not at all
2
3
4
5
Perfectly
24
Q

In which of these cases does the filing date stay unchanged
even though parts of the application are filed after receipt of the filing documents?

A. Missing drawings are filed six weeks after invitation by the EPO to file the missing parts.

B. Missing drawings are filed within the applicable time limit and some of the drawings were disclosed in the priority document.

C. Missing drawings are filed later and the criteria of Rule 56(3)
EPC are met, but priority is validly claimed only after filing of the application.

D. Missing drawings are filed later than two months after invitation by the EPO to file the missing parts.

A

False, False, True, True

See Rule 56 EPC

How well did you know this?
1
Not at all
2
3
4
5
Perfectly
25
Q

Are the following statements concerning the physical requirements true or false?

A. Drawings may be coloured.
B. Sheets should be used on one side only, with numbers to be
placed at the top centre of the page.
C. Line numbering in sets of five should be used.
D. Typing must be 1.5 spaced.

A

False (see Rule 46(2)(a) EPC), True, True, True

See Rule 46 EPC

How well did you know this?
1
Not at all
2
3
4
5
Perfectly
26
Q

A European patent application does not contain an abstract. The formalities officer communicates the absence of the abstract. Three months after communication, the abstract is filed. True or false?

A. The application is deemed to be withdrawn.
B. The application will not meet the formal requirements, because it is submitted too late.
C. The abstract will be used, but the applicant has to pay a late
filing fee.
D. The abstract of the priority document will be used.

A

False, True, False, False

According to Rule 57(d) EPC, the application is checked for an
abstract. If it is missing, an invitation according to Rule 58 EPC with
a two-month time limit is communicated. The legal consequence of
not filing the abstract within this time limit cannot be found in Rule 58
EPC. Rule 58 EPC implements Article 90 EPC, where the legal
consequence can be found. Article 90(5) EPC states that the
application will be refused.

How well did you know this?
1
Not at all
2
3
4
5
Perfectly
27
Q

Examination as to formal requirements - Rule 57 (a)

A

If the European patent application has been accorded a date of filing, the European Patent Office shall examine, in accordance with Article 90, paragraph 3, whether:

(a) a translation of the application required under Article 14, paragraph 2, under Rule 36, paragraph 2, second sentence, or under Rule 40, paragraph 3, second sentence, has been filed in due time;

How well did you know this?
1
Not at all
2
3
4
5
Perfectly
28
Q

Examination as to formal requirements - Rule 57 (b)

A

If the European patent application has been accorded a date of filing, the European Patent Office shall examine, in accordance with Article 90, paragraph 3, whether:

(b) the request for grant of a European patent satisfies the requirements of Rule 41;

How well did you know this?
1
Not at all
2
3
4
5
Perfectly
29
Q

Examination as to formal requirements - Rule 57 (c)

A

If the European patent application has been accorded a date of filing, the European Patent Office shall examine, in accordance with Article 90, paragraph 3, whether:

(c) the application contains one or more claims in accordance with Article 78, paragraph 1(c), or a reference to a previously filed application in accordance with Rule 40, paragraphs 1(c), 2 and 3, indicating that it replaces also the claims;

How well did you know this?
1
Not at all
2
3
4
5
Perfectly
30
Q

Examination as to formal requirements - Rule 57 (d)

A

If the European patent application has been accorded a date of filing, the European Patent Office shall examine, in accordance with Article 90, paragraph 3, whether:

(d) the application contains an abstract in accordance with Article 78, paragraph 1(e);

How well did you know this?
1
Not at all
2
3
4
5
Perfectly
31
Q

Examination as to formal requirements - Rule 57 (e)

A

If the European patent application has been accorded a date of filing, the European Patent Office shall examine, in accordance with Article 90, paragraph 3, whether:

(e) the filing fee and the search fee have been paid in accordance with Rule 17, paragraph 2, Rule 36, paragraph 3, or Rule 38;

How well did you know this?
1
Not at all
2
3
4
5
Perfectly
32
Q

Examination as to formal requirements - Rule 57 (f)

A

If the European patent application has been accorded a date of filing, the European Patent Office shall examine, in accordance with Article 90, paragraph 3, whether:

(f) the designation of the inventor has been made in accordance with Rule 19, paragraph 1;

How well did you know this?
1
Not at all
2
3
4
5
Perfectly
33
Q

Examination as to formal requirements - Rule 57 (g)

A

If the European patent application has been accorded a date of filing, the European Patent Office shall examine, in accordance with Article 90, paragraph 3, whether:

(g) where appropriate, the requirements laid down in Rules 52 and 53 concerning the claim to priority have been satisfied;

How well did you know this?
1
Not at all
2
3
4
5
Perfectly
34
Q

Examination as to formal requirements - Rule 57 (h)

A

If the European patent application has been accorded a date of filing, the European Patent Office shall examine, in accordance with Article 90, paragraph 3, whether:

(h) where appropriate, the requirements of Article 133, paragraph 2, have been satisfied;

How well did you know this?
1
Not at all
2
3
4
5
Perfectly
35
Q

Examination as to formal requirements - Rule 57 (i)

A

If the European patent application has been accorded a date of filing, the European Patent Office shall examine, in accordance with Article 90, paragraph 3, whether:

(i) the application meets the requirements laid down in Rule 46 and Rule 49, paragraphs 1 to 9 and 12;

How well did you know this?
1
Not at all
2
3
4
5
Perfectly
36
Q

Examination as to formal requirements - Rule 57 (j)

A

If the European patent application has been accorded a date of filing, the European Patent Office shall examine, in accordance with Article 90, paragraph 3, whether:

(j) the application meets the requirements laid down in Rule 30.

How well did you know this?
1
Not at all
2
3
4
5
Perfectly
37
Q

A European patent application comprises 45 pages and 14
claims. True or False, the following fees have to be paid within one month after filing?

A. Claims fee.
B. Search fee.
C. Designation fee.
D. Filing fee.
E. Additional filing fee for excess pages.
A

False, True, False, True, True

See Rule 58 EPC

How well did you know this?
1
Not at all
2
3
4
5
Perfectly
38
Q

True or false?
A. During examination proceedings an English translation of an
application originally filed in Japanese can be corrected if the
translation was not accurate.
B. If completely incorrect documents were filed by mistake, this can be corrected under Rule 139 EPC.
C. Drawings should not include text.
D. Any third party can pay due fees by bank transfer.

A

True, False, True, True

See Rule 139 EPC

How well did you know this?
1
Not at all
2
3
4
5
Perfectly
39
Q

A Japanese scientist having his residence in Tokyo has written an article for a Japanese scientific journal. The article describes an invention which he would like to protect. The article is in Japanese. As the journal containing the article is to be published tomorrow, he faxes the article to the EPO today, 6 March 2012, together with a cover sheet indicating in English that a European patent is sought. He gives only his name and his email address.

Will the faxed article be considered a European patent
application and be accorded a date of filing?

A

Anybody can file a European patent application (Article 58 EPC). It is possible to file a European patent application in Japanese (Article 14(2) EPC).

The fax contains an indication that a European patent is sought, so Rule 40(1)(a) EPC is satisfied.

A fax response number or a name and an email address are sufficient for identification if they “allow the applicant to be contacted” (Rule 40(1)(b) EPC; GL A-II, 4.1).

The article is considered a description; only a description is required (claims are not required for accordance of a date of filing) (Rule 40(1)(c) EPC).

Filing by means of fax is possible (Rule 2(1) EPC; GL A-II, 1.2.1; OJ EPO 2019, A18).

All the requirements of Rule 40 EPC are fulfilled; according to Article 80 EPC, a date of filing will be accorded.

How well did you know this?
1
Not at all
2
3
4
5
Perfectly
40
Q

A Japanese scientist having his residence in Tokyo has written an article for a Japanese scientific journal. The article describes an invention which he would like to protect. The article is in Japanese. As the journal containing the article is to be published tomorrow, he faxes the article to the EPO today, 6 March 2012, together with a cover sheet indicating in English that a European patent is sought. He gives only his name and his email address.

What formal requirements have to be fulfilled for the EPO to start on the search, and within

A

The formal requirements to be met are as follows:

Appoint a representative (Article 133(2) EPC, Rule 57(h) EPC) within two months from an invitation from the EPO (Rule 58 EPC).

File a translation into an official language of the EPO (Article 14(2) EPC) within two months of filing the application (Rule 6(1) EPC): 06.03.2012 + two months (Rule 131(4) EPC) →06.05.2012
(Sunday) (Rule 134(1) EPC) → 07.05.2012 (Monday), or within two months from an invitation from the EPO (Rule 57(a) and Rule 58 EPC).

Pay the filing fee and the search fee within one month of filing (Article 78(2) and Rule 38(1) EPC). Note: Where the application contains more than 35 pages, an additional page fee will have to be paid (Rule 38(2) EPC; Article 2, item 1a, RFees). 06.03.2012 + one month (Rule 131(4) EPC) → 06.04.2012 (Good Friday) (Rule 134(1) EPC) → 10.04.2012 (Tuesday).

Use the Request for grant form (EPO Form 1001) in order to fulfil the requirements of Rule 41 EPC within two months of an invitation from the EPO (Rule 57(b) EPC; Rule 58 EPC). File claims based on the description within two months of an invitation from the EPO (Article 78(1)(c), Rules 57(c) and 58 EPC); possibly pay a surcharge for claims fees under Rule 45(2)
EPC.

File an abstract according to Article 78(1)(e) EPC within two months of an invitation from the EPO (Article 78(1)(e), Rules 57(d) and 58 EPC).

How well did you know this?
1
Not at all
2
3
4
5
Perfectly
41
Q

A European patent application is filed by a Japanese applicant living in Japan by reference to the description and drawings of an earlier Japanese patent application.

Will a filing date be accorded to the application?

A

Filing date: When the European patent application was filed, the requirements for accordance of a filing date were fulfilled (Article 80 and Rule 40(1) EPC). Assuming the requirements of Rule 40(1)(a) EPC (indication that a European patent is sought) and Rule 40(1)(b) EPC (information identifying the applicant) to have been fulfilled upon filing of the European patent application, the European patent application will be accorded a filing date, because for this it is sufficient if reference is made to a previously filed application (Rule 40(1)(c) EPC). A Japanese national resident in Japan need not be represented for the purposes of filing a European patent application (Articles 58 and 133(1) and (2) EPC).

How well did you know this?
1
Not at all
2
3
4
5
Perfectly
42
Q

A European patent application is filed by a Japanese applicant living in Japan by reference to the description and drawings of an earlier Japanese patent application.

What requirements must be fulfilled when reference is made to a previously filed application?

A

Required information: Where the description is filed by reference to a previously filed application, that reference must contain the following information in order for the European patent application to qualify for a filing date (Rule 40(2) EPC; GL A-II, 4.1.3.1):

▪ the filing date
▪ the file number
▪ the office with which it was filed
▪ an indication that this reference replaces the description and
any drawings

Copy of the previously filed application
In addition, the applicant must supply a certified copy of the previously filed application within two months of the filing date (Rule 40(3) EPC, first sentence).

However, according to Rule 40(3), last sentence, EPC this requirement is waived where the previously filed application is already available to the EPO under the conditions specified in OJ EPO 2009, 486, that is, where the previously filed application is a Euro-direct application or an international application filed with the EPO as receiving office. In this case, a certified copy is required. See also GL A-II, 4.1.3.1.

Translation of the previously filed application
If the previously filed application is not in an official language of the EPO, the applicant must also file a translation into one of the official EPO languages within two months of the filing date (Rule 40(3), second sentence, EPC; GL A-II, 4.1.3.1). If the translation of the previously filed application is already available to the EPO, a copy of it will be included in the file free of charge and the applicant is not required to file it (Rule 40(3) EPC).

Representation
All the above acts can be performed by the Japanese applicant upon filing the European patent application.

However, the applicant must be represented by a professional representative and act through him in all proceedings after filing the European patent application (Article 133(2) EPC)

How well did you know this?
1
Not at all
2
3
4
5
Perfectly
43
Q

A European patent application is filed by a Japanese applicant living in Japan by reference to the description and drawings of an earlier Japanese patent application.

Can the applicant also refer to the claims in the Japanese
application, or must a separate set of claims be filed? If so, by
when?

A

Claims: The applicant can indicate that he wishes the claims of the previously filed application to take the place of the claims in the application as filed (see Rule 57(c) EPC).

Such an indication must be made on the filing date, preferably by crossing the appropriate box on the Request for grant form (EPO Form 1001; GL A-II, 4.1.3.1).

However, if the applicant does not refer to the claims of the previously filed application, but only to the description and any drawings thereof, he may, at the same time as filing the reference (i.e. on the filing date), file a set of claims.

If the applicant does not file a set of claims on filing the European patent application, he can do so of own volition (shortly) after filing the application (GL A-III, 9, bullet point (iii)).

For all acts after filing the European patent application, the Japanese applicant must be represented.

How well did you know this?
1
Not at all
2
3
4
5
Perfectly
44
Q

A European patent application is filed by a Japanese applicant living in Japan by reference to the description and drawings of an earlier Japanese patent application.

What happens if the applicant does not refer to the claims in the Japanese application and does not file a set of claims?

A

No claims: A European patent application requires at least one claim (Article 78(1)(c) EPC).

If the applicant does not refer to the claims in the Japanese application and does not file a set of claims of his own volition (shortly) after filing the application, the EPO, upon checking the formal requirements (Article 90(3) and Rule 57(c) EPC), will send an invitation to file at least one claim within a period of two months (Article 90(4) and Rule 58 EPC).

If the applicant fails to file the claims in due time in response to the invitation (Rule 58 EPC), the European patent application will be refused (Article 90(5) EPC).

For all acts after the filing of the European patent application, the Japanese applicant must be represented.

How well did you know this?
1
Not at all
2
3
4
5
Perfectly
45
Q

A European patent application is filed by a Japanese applicant living in Japan by reference to the description and drawings of an earlier Japanese patent application.

Are there remedies if the applicant does not comply with the
invitation from the EPO?

A

Remedies: Further processing is excluded in the period of Rule 58 EPC (Article 121(4) and Rule 135(2) EPC).

However, re-establishment of rights can be requested if all due care can be proven (Article 122 and Rule 136 EPC).

How well did you know this?
1
Not at all
2
3
4
5
Perfectly
46
Q

A French citizen resident in Japan files a new European patent application by post with the EPO.

Does he need to appoint a representative to do this?

A

No. A representative is not required for the mere act of filing a European patent application (Article 58 EPC).

How well did you know this?
1
Not at all
2
3
4
5
Perfectly
47
Q

Having filed the application, is this same French citizen now required to appoint a representative?

A

Yes. He has to appoint a representative now because he is not resident in a contracting state (Article 133(2) EPC).

How well did you know this?
1
Not at all
2
3
4
5
Perfectly
48
Q

An Australian applicant (not resident or having their principal place of business in an EPC contracting state) wishes to seek further processing under Article 121 EPC. Does he have to appoint a representative?

A

Yes. However, the further processing fee can be paid by anyone (Article 133(2) EPC and GL A-X, 1: Fees may be validly paid by any person).

How well did you know this?
1
Not at all
2
3
4
5
Perfectly
49
Q

Opposition has been filed by a resident of the USA who has signed the notice of opposition himself. Is the opposition admissible?

(a) Yes, if a professional representative is appointed within a time limit set by the EPO.
(b) No.

A

Answer A is correct. The notice of opposition must be approved by the subsequently appointed representative.

How well did you know this?
1
Not at all
2
3
4
5
Perfectly
50
Q

The applicant for a European patent application is a Japanese citizen resident in France. Is he required to appoint a representative?

A

No. Persons resident in a contracting state do not need to be represented (Article 133(1) EPC).

How well did you know this?
1
Not at all
2
3
4
5
Perfectly
51
Q

A United States attorney has filed a patent application with the EPO on behalf of a client. Can the US attorney act as a professional representative for the client before the EPO?

A

No. Article 134(8) EPC permits only legal practitioners qualified in a contracting state to act as professional representatives.

How well did you know this?
1
Not at all
2
3
4
5
Perfectly
52
Q

A European patent application is assigned from a German applicant to a United States citizen. What essential step must the new applicant take?

A

The new applicant (US citizen) must authorise a representative in order to comply with Article 133(2) EPC.

How well did you know this?
1
Not at all
2
3
4
5
Perfectly
53
Q

A European patent application is filed by joint applicants, one of whom is resident in Germany, the other in Bosnia and Herzegovina. Do they have to appoint a representative

A

Yes. Bosnia and Herzegovina is an extension state, not an EPC contracting state, so the resident there must appoint a representative (Article 133(2) EPC), who will then become the common representative, unless the German applicant has appointed a representative (Rule 151(1) EPC).

How well did you know this?
1
Not at all
2
3
4
5
Perfectly
54
Q

A European patent application in Japanese is filed by a German resident who appoints a representative on the “Request for grant” form. The EPO writes to the applicant to invite him to file a translation within two months. The applicant takes no action. What will happen?

A

Nothing. The EPO should have written to the appointed representative in accordance with Rule 130(1) EPC.

How well did you know this?
1
Not at all
2
3
4
5
Perfectly
55
Q

Are persons accompanying representatives at oral proceedings entitled to make submissions as a matter of course?

A

No. They can only do so with the permission and at the

discretion of the EPO (G 4/95)

How well did you know this?
1
Not at all
2
3
4
5
Perfectly
56
Q

A request for examination of a European patent application is filed by a person who is not entitled to act as a representative under Article 134 EPC but who is subsequently entered on the list of professional representatives. Was the request for examination validly filed? (You may wish to read decision J 28/86 before answering this question).

A

No. Being entered on the list of professional representatives (Article 134(1) EPC) does not validate acts performed by a person who was not entitled to do so at the time (J 28/86).

How well did you know this?
1
Not at all
2
3
4
5
Perfectly
57
Q

Three applicants file a joint European application. No common representative has been appointed. The first, second and third applicants named on the request for grant are designated Applicants 1, 2 and 3 respectively.

(a) All three applicants are from an EPC contracting state. Only Applicant 3 has appointed a professional representative. Is this professional representative the common representative for all three applicants?
(b) Applicants 1 and 3 are from a contracting state, while Applicant 2 is a Japanese company. No professional representative has been appointed. Is Applicant 1 the deemed common representative?
(c) Applicants 1 and 3 are from a contracting state, while Applicant 2 is a Japanese company. Each applicant has appointed a separate professional representative. Is the representative of Applicant 1 the deemed common representative?
(d) Applicants 1 and 3 are from Germany, while Applicant 2 is a company from Montenegro. Only Applicant 3 has appointed a professional representative. Is the representative of Applicant 1 the deemed common representative?

A

(a) No. Since all the applicants reside/have their place of business in a contracting state, they are not obliged to appoint a professional representative (Article 133(1) EPC). In this situation it is the (representative of the) first-named applicant who is the deemed common representative (Rule 151(1), first sentence, EPC).
(b) No. Applicant 2 has to appoint a professional representative (Article 133(2) EPC), who will be the deemed common representative (Rule 151(1), second sentence, EPC).
(c) Yes. See Rule 151(1), second sentence, EPC.
(d) No. Applicant 2 has to be represented, since Montenegro is an extension state and not a contracting state (Article 133(2) EPC). According to Rule 151(1), second sentence, EPC Applicant 2’s professional representative will become the deemed common representative

How well did you know this?
1
Not at all
2
3
4
5
Perfectly
58
Q

A Spanish national living in Japan wants to file a European application in Spanish at the EPO, claiming the priority of an earlier Spanish application. Does he need a professional representative for the following actions:

(a) Filing a European application in Spanish at the EPO.
(b) Paying the filing fee and search fee.
(c) Filing a translation in French three weeks after the filing date.
(d) Providing a copy of the priority documents, five days after the filing date.

A

(a) No. Filing can be performed by any applicant (Article 133(2) EPC), regardless of where they are located, and in any language (Article 14(2) EPC).
(b) No. Payments may be made by any person (GL A-X, 1), as they do not constitute an act.

(c) Yes. The applicant does not have his residence or place of business in a contracting state (since he is in Japan) and thus Article 133(2) EPC applies. The fact that he is a Spanish national is of no help, since only the residence or place of business – and not the nationality – is relevant for determining the need for
compulsory representation (Article 133(2) EPC). A date of filing has already been accorded; all subsequent acts have to be performed by a professional representative. The subsequent filing of a French translation must be done through a professional representative.

(d) Yes. Since this is a document filed after the filing of the European patent application, according to Article 133(2) EPC a professional representative is required to submit the certified copy of the Spanish priority document.

How well did you know this?
1
Not at all
2
3
4
5
Perfectly
59
Q

Company A has its principal place of business in France. Which of the following are allowed to represent it before the EPO?

(a) A French patent attorney.
(b) A German “Rechtsanwalt” (legal practitioner).
(c) An employee of an economically connected French company B who is not a professional representative.
(d) An authorised employee of company A of Japanese nationality.

A

(a) No. A national patent attorney is not considered to be a legal practitioner (J 19/89).
(b) Yes. See Article 134(8) EPC.
(c) No. An employee is not allowed to act for an economically connected company (T 298/97).
(d) Yes. See Article 133(3) EPC. There is no restriction regarding the nationality of the employee. The person need only be employed by a company having its principal place of business in a contracting state.

How well did you know this?
1
Not at all
2
3
4
5
Perfectly
60
Q

Which of the following must file a signed authorisation in order to act before the EPO?

(a) Legal practitioners.
(b) Professional representatives.
(c) Company employees.
(d) Professional representatives employed at a company.

A

Yes, No, Yes, No

How well did you know this?
1
Not at all
2
3
4
5
Perfectly
61
Q

Will competitors know if an applicant files a PACE request for accelerated examination online using Form 1005?

A

No. The EPO does not publish PACE requests. Moreover, by decision of the President dated 12 July 2007, they are excluded from file inspection (Ancillary Regulations to the EPC 2016, 281-282, Article 1(c); OJ EPO 2015, A93).

How well did you know this?
1
Not at all
2
3
4
5
Perfectly
62
Q

Is a pending infringement action in respect of a European patent a circumstance that justifies opposition being dealt with rapidly?

A

Yes. Requests for accelerated processing may be filed at any time in cases where an infringement action in respect of a European patent is pending before a national court of a contracting state (Notice from the EPO dated 17 March 2008, OJ EPO 2008, 221; GL E-III, 5). Such requests are not part of the PACE programme.

How well did you know this?
1
Not at all
2
3
4
5
Perfectly
63
Q

Can a PACE request filed during examination of an application be extended by the patent proprietor if an opposition is filed? What acts are required, if any?

A

No. There is no such provision under the PACE programme. PACE is a programme for the accelerated processing of patent applications. However, acceleration of opposition proceedings is possible. Any party to the opposition proceedings desiring accelerated processing must file written arguments in support of their request (Notice from the EPO dated 17 March 2008, OJ EPO 2008, 221; GL E-VIII, 5).

How well did you know this?
1
Not at all
2
3
4
5
Perfectly
64
Q

Having filed a request for accelerated examination, an applicant receives a communication under Article 94(3) EPC. As his workload is very heavy, he realises that he will not be able to meet the time limit set in the communication, so he requests an extension under Rule 132(2). Will the requested extension affect the processing of his application under PACE, and if so, how?

A

Yes. The application will be removed from the PACE programme since the applicant has requested an Extension of a time limit (OJ EPO 2015, A93, item 4).

How well did you know this?
1
Not at all
2
3
4
5
Perfectly
65
Q

One week after filing a Euro-PCT application, where the ISA was the USPTO, an applicant requests accelerated examination under PACE. Will the examination of the application be processed under PACE?

A

No. Accelerated examination can only be requested once responsibility for the application has passed to the examining division (OJ EPO 2015, A93, points 2 and 12). The ISA was the USPTO, so the EPO will have to draw up a supplementary search report, and it is only when the applicant confirms his request for examination following the Rule 70(2) EPC invitation that the examining division becomes responsible (Rule 10(3) EPC; GL C-II, 1). Therefore, a PACE request filed with an application will not result in accelerated examination under PACE, because at the time the Euro-PCT application was filed, the examining division was not responsible.

How well did you know this?
1
Not at all
2
3
4
5
Perfectly
66
Q

You are a European patent attorney authorised using Form 1003 to represent Client A in matters relating to European patent application EP-1. The time limit for requesting examination expired last week and you did nothing, having received no instructions. Today you learn that your client passed away two weeks before the time limit expired. You are unable to obtain instructions because it is not yet clear who the rightful heir is to EP-1. How would you recommend to proceed?

A

The authorisation on Form 1003 does not specify that it terminates upon the death of the authorisor (Rule 152(9) EPC). The proceedings are therefore not automatically interrupted (Rule 142(1)(a) 2nd sentence). You should inform the EPO that the applicant has passed away and obtain an interruption of proceedings pursuant to Rule 142 EPC. Proceedings will be interrupted from the date the EPO received this
request (GL E-VII, 1.3), which should be before the expiry of the time limit for requesting further processing expires.

Upon resumption, of the proceedings, you would then have to request further processing (Article 121 and Rule 135 EPC), as further processing is not ruled out for the period under Rule 70(1) EPC (GL A-VI, 2.3).

How well did you know this?
1
Not at all
2
3
4
5
Perfectly
67
Q

True or false? If application proceedings before the EPO are interrupted due to the death or legal incapacity of the authorised representative of an applicant not subject to mandatory representation under Article 133(2) EPC, proceedings will be resumed:

Three months after the start of the interruption.

A

Where the interruption is caused by circumstances concerning the representative (Rule 142(1)(c) EPC), proceedings are resumed when the EPO is informed of the new representative (Rule 142(3) EPC). However, if no new representative has been appointed by an applicant who is not subject to mandatory representation within three months of the beginning of the interruption, the EPO will send a communication to the applicant stating that the proceedings will be resumed as from the date of notification of the communication (Rule 142(3)(b) EPC)

False. Proceedings are resumed either after a new representative has been appointed or from the date of notification of a communication by the EPO to the applicant or proprietor under Rule 142(3) (b) EPC. The EPO sends the communication if a new representative has not been appointed within three months. So three months is the period after which the EPO sends the communication, but proceedings are not resumed until notification (as per Rule 126(2) EPC) of that communication.

How well did you know this?
1
Not at all
2
3
4
5
Perfectly
68
Q

True or false? If application proceedings before the EPO are interrupted due to the death or legal incapacity of the authorised representative of an applicant not subject to mandatory representation under Article 133(2) EPC, proceedings will be resumed:

If the EPO is informed within three months from the beginning of the interruption that a new representative has been appointed, at the time the EPO is advised of the new representative.

A

Where the interruption is caused by circumstances concerning the representative (Rule 142(1)(c) EPC), proceedings are resumed when the EPO is informed of the new representative (Rule 142(3) EPC). However, if no new representative has been appointed by an applicant who is not subject to mandatory representation within three months of the beginning of the interruption, the EPO will send a communication to the applicant stating that the proceedings will be resumed as from the date of notification of the communication (Rule 142(3)(b) EPC)

True. Proceedings are resumed when the EPO is informed of the new representative (Rule 142(3) EPC) if the new representative is appointed within three months of the start of the interruption. If a new representative has not been appointed within three months, the EPO will send a communication under Rule 142(3)(b) EPC.

How well did you know this?
1
Not at all
2
3
4
5
Perfectly
69
Q

True or false? If application proceedings before the EPO are interrupted due to the death or legal incapacity of the authorised representative of an applicant not subject to mandatory representation under Article 133(2) EPC, proceedings will be resumed:

If the EPO is not advised of the new representative within three months of the start of the interruption, from the date of notification of the EPO communication stating that the proceedings will be resumed.

A

Where the interruption is caused by circumstances concerning the representative (Rule 142(1)(c) EPC), proceedings are resumed when the EPO is informed of the new representative (Rule 142(3) EPC). However, if no new representative has been appointed by an applicant who is not subject to mandatory representation within three months of the beginning of the interruption, the EPO will send a communication to the applicant stating that the proceedings will be resumed as from the date of notification of the communication (Rule 142(3)(b) EPC)

True. See Rule 142(3)(b) EPC.

How well did you know this?
1
Not at all
2
3
4
5
Perfectly
70
Q

True or false? If application proceedings before the EPO are interrupted due to the death or legal incapacity of the authorised representative of an applicant not subject to mandatory representation under Article 133(2) EPC, proceedings will be resumed:

If the EPO is not advised of the new representative within three months of the start of the interruption, two months from notification of the EPO communication stating that the proceedings will be resumed

A

Where the interruption is caused by circumstances concerning the representative (Rule 142(1)(c) EPC), proceedings are resumed when the EPO is informed of the new representative (Rule 142(3) EPC). However, if no new representative has been appointed by an applicant who is not subject to mandatory representation within three months of the beginning of the interruption, the EPO will send a communication to the applicant stating that the proceedings will be resumed as from the date of notification of the communication (Rule 142(3)(b) EPC)

False. Proceedings are resumed in this scenario (where the applicant is not subject to mandatory representation) from the date of notification of the EPO communication (Rule 142(3)(b) EPC) and not two months from the date of notification of such communication.

How well did you know this?
1
Not at all
2
3
4
5
Perfectly
71
Q

True or false? When EPO proceedings are resumed after an interruption:

(a) All time limits which were running at the time of interruption start again.
(b) All time limits which were running at the time of interruption start again, except for the time limits for requesting examination and for paying renewal fees.
(c) The time limit for examination does not begin again but continues for at least two months.
(d) The time limit for examination does not begin again but continues for at least three months.

A

(a) False. Upon resumption of proceedings, the time limits for requesting examination and paying renewal fees do not begin again (Rule 142(4) EPC)
(b) True. The time limits for requesting examination and paying renewal fees are exceptions to the general principle that any time limits in force at the start of the interruption begin again as from the day of resumption (Rule 142(4) EPC).
(c) True. The time limit for requesting examination does not begin again on resumption of the proceedings, but will be suspended, i.e. it does not expire during the interruption, and continues for at least two months after resumption (Rule 142(4) EPC; J 7/83).
(d) False. The time limit for requesting examination continues for at least two (not three) months.

How well did you know this?
1
Not at all
2
3
4
5
Perfectly
72
Q

True or false? Proceedings can be interrupted under Rule 142 EPC in the event of:

(a) The death or legal incapacity of the applicant.
(b) The death or legal incapacity of the proprietor.
(c) The death or legal incapacity of the opponent.
(d) The death or legal incapacity of the representative.

A

(a) True. Proceedings can be interrupted in the event of the death or legal incapacity of the applicant (Rule 142(1) (a) EPC).
(b) True. Proceedings can be interrupted in the event of the death or legal incapacity of the proprietor (Rule 142(1) (a) EPC).
(c) False. The provisions of Rule 142 EPC only deal with cases where the applicant or proprietor is hindered from performing legal acts. They cannot be used by an opponent to interrupt proceedings. In the event of the death or legal incapacity of an opponent, see Rule 84(2) EPC.
(d) True. Proceedings will be interrupted in the event of the death or legal incapacity of the applicant or proprietor’s representative (Rule 142(1) (c) EPC).

73
Q

You are a European patent attorney. Your client, B, was legally incapacitated by illness for almost a year, but is now well again. Client B has two European patent applications, EP-1 and EP-2. Because of his illness, proceedings have been interrupted by the EPO for EP-1 and EP-2. Ten months ago, during B’s illness, and while the proceedings were interrupted, the time limit for requesting examination expired for EP-1. A
communication from the examining division setting a four-month time limit for reply was issued in respect of EP-2 while B was legally incapacitated, but one month before the EPO was informed of the illness with the request for interruption of proceedings. The communication set a period of four months to respond.

When will the time limit for requesting examination for EP-1 expire?

A

Two months after the resumption of proceedings (Rule 142(4) EPC, second sentence).

74
Q

You are a European patent attorney. Your client, B, was legally incapacitated by illness for almost a year, but is now well again. Client B has two European patent applications, EP-1 and EP-2. Because of his illness, proceedings have been interrupted by the EPO for EP-1 and EP-2. Ten months ago, during B’s illness, and while the proceedings were interrupted, the time limit for requesting examination expired for EP-1. A
communication from the examining division setting a four-month time limit for reply was issued in respect of EP-2 while B was legally incapacitated, but one month before the EPO was informed of the illness with the request for interruption of proceedings. The communication set a period of four months to respond.

When will the time limit for responding to the communication for EP-2 expire?

A

Four months after the resumption of proceedings (Rule 142(4) EPC, second sentence). The proceedings are interrupted as from the date of receipt of the request at the EPO, which is one month before the expiry of the four-month time limit. However, Rule 142(4) EPC states that the four-month time limit will begin again, which is why the answer is four months after resumption of proceedings.

75
Q

True or false? When proceedings for grant are stayed due to the institution of entitlement proceedings within the meaning of Article 61 EPC, upon resumption of proceedings:

All time limits begin again in all circumstances.

A

False. In the event of a stay of proceedings under Rule 14 EPC, upon resumption of the proceedings none of the time limits begin again. Rule 14(4) EPC provides that all periods other than those for the payment of renewal fees which are running at the date of the stay of proceedings will be interrupted by such stay. The time which has not yet elapsed will begin to run from the date on which proceedings are resumed. However, the time still to run after such
resumption will not be less than two months.

76
Q

True or false? When proceedings for grant are stayed due to the institution of entitlement proceedings within the meaning of Article 61 EPC, upon resumption of proceedings:

The time which has not yet elapsed will begin to run from the date on which proceedings are resumed, in all circumstances. There is no minimum time limit.

A

False. The period for the payment of renewal fees keeps running (Rule 14(4) EPC). Furthermore, the time still to run, other than for the payment of renewal fees, will not be less than two months.

77
Q

True or false? When proceedings for grant are stayed due to the institution of entitlement proceedings within the meaning of Article 61 EPC, upon resumption of proceedings:

The time which has not yet elapsed will begin to run from the date when proceedings are resumed, in all
circumstances other than for renewal fee payments. There is no minimum time limit.

A

False. The time still to run, other than for the payment of renewal fees, will not be less than two months (Rule 14(4) EPC).

78
Q

True or false? When proceedings for grant are stayed due to the institution of entitlement proceedings within the meaning of Article 61 EPC, upon resumption of proceedings:

The time which has not yet elapsed will begin to run from the date when proceedings are resumed, in all circumstances other than for renewal fee payments. However, the time limit will not be less than two months.

A

True. Rule 14(4) EPC stipulates that, other than for the payment of renewal fees, the time still to run after such resumption is not less than two months.

79
Q

Your client C has been found, in the competent national court of law, to be the rightful applicant for EP-3, which was filed by D two years ago. You have informed the EPO of this, and the EPO has informed you that proceedings will be resumed on 3 February 2020.

Proceedings were stayed on 4 June 2019. On 6 May 2019 the examining division had issued a communication giving a four-month period to respond. What will be the new time limit for the response?

A

15 May 2020. The communication is deemed to have been notified on 16 May 2019 (10-day rule, Rule 126(2) EPC) and the period would normally have ended on 16 September 2019.

There were still 13 days left of the first month plus three months of the four-month time limit when the proceedings were stayed on 4 June 2019 (i.e. 4, 5, 6, 7, 8, 9, 10, 11, 12, 13, 14, 15, 16 June equals 13 days and July, August, September equals three months).

According to Rule 14(4) EPC, the period should begin to run from the date on which proceedings are resumed, and should not be less than two months.

The period starts running again on 3 February 2020, which counts as the first of the 13 days. The period therefore ends on 15 May 2020.

80
Q

Which of the following statements is correct?

(a) Any applicant adversely affected by a decision of the EPO may request further processing.
(b) If an applicant fails to reply within a time limit vis-à-vis the EPO, he may always request further processing.
(c) The fee for further processing is EUR 255 or 50% of the late fee (where a fee has been missed).
(d) Further processing may be requested where the applicant has missed the time limit for entry into the national phase under the PCT.

A

(a) False. See Article 121(4) and Rule 135(2) EPC.
(b) False. Not all missed time limits can be remedied by further processing. A number of situations are excluded in Article 121(4) EPC and Rule 135(2) EPC.
(c) False. See Article 2(1)(12) RFees. The flat fee increased in 2020 to EUR 265.
(d) True. See PCT Applicant’s Guide, National Phase, Annex EP (as designated or elected Office), paragraph EP.25

81
Q

Which of the following requests for further processing can be granted?

(a) Missing the time limit for filing a translation under Article 14(2) EPC.
(b) Missing the time limit for responding to a communication from the EPO examining division inviting the applicant to correct deficiencies under Rule 71(1) EPC.
(c) Failure to timely pay filing, examination and/or designation fees in a divisional application.
(d) Missing the time limit for filing the translation of a priority document during examination proceedings if not filed in an official language of the EPO.
(e) Failure to pay grant and printing fees and filing the translation of claims upon grant.

A

(a) False. Further processing is ruled out for the filing of a translation under Article 14(2) EPC by Rule 135(2) EPC. Rule 135(2) EPC rules out the period under Rule 6(1) EPC, which in turn refers to translations under Article 14(2) EPC. Please note that you have to first find the specific rule or article that makes reference to the
time limit, and then check if that article or rule is
excluded from further processing.

(b) True. A time limit is set in any communication under Rule 71(1) EPC, and failure to reply in due time means that the application is deemed to be withdrawn (Article 94(4) EPC). Further processing is the remedy, as it is not excluded in Rule 135(2) and Article 121(4) EPC.
(c) True. See GL A-IV, 1.4.1.
(d) True. See GL A-III, 6.8.3.
(e) True. Failure to observe the time limit for payment of grant and printing fees and filing the translation under Rule 71(3) EPC leads to the application being deemed to be withdrawn under Rule 71(7) EPC. Further processing is the remedy, as it is not excluded in Rule 135(2) and Article 121(4) EPC.

82
Q

An applicant misses the time limit for replying to a communication pursuant to Article 94(3) EPC from the EPO. Upon receipt of the noting of loss of rights pursuant to Rule 112(1) EPC, he completes the omitted act within the permitted period, but having no money, asks a friend to pay the further processing fee. Will further processing be allowed? Please provide legal basis to support your answer. If you would like a hint, please see the footnote below.

A

Yes. Further processing is not ruled out for failure to respond to communications pursuant to Article 94(3) EPC. The requirement is that the omitted act (the response) be completed and that the fee for further processing be paid. It is not a requirement for the representative or applicant to pay the further processing fee, as fees may be validly paid by any person (GL A-X, 1).

83
Q

An applicant requests further processing in a letter to the EPO, pays the further processing fee and completes the omitted act. Will further processing be allowed?

A

Yes. Although it is not necessary to submit the request by letter, this will not cause the EPO to refuse the request.

84
Q

In a communication under Article 94(3) EPC dated 17 May 2019, the EPO set the applicant a time limit of four months to reply to an objection of lack of novelty. On 18 September 2019 , the applicant calculated that he had missed the deadline for reply and requested further processing by paying the further processing fee and completing the omitted act. Will further processing be allowed?

A

No. The time limit for reply to the lack of novelty objection does not expire until 27 September 2019, so there has been no loss of rights. The letter dated 17 May 2019 is deemed to be notified (Rule 126(2) EPC) 10 days later on 27 May 2019, on which date the four month time limit starts, which therefore expires on 27 September 2019. The EPO will refund the fee as lacking legal basis (GL A-X, 10.1.1).

85
Q

An applicant not using Form 1200 has missed the time limit for requesting examination under Rule 159(1)(f) EPC (entry from the international phase of the PCT into the European regional phase). Article 94(1) EPC states that, in addition to a request for examination, the examination fee must be paid. The applicant requests further processing. Does he have to pay two further processing fees, one in respect of the request for examination and another in respect of the examination fee? Please provide legal basis to support your answer.

A

Yes. Payment of the examination fee is a separate act from submission of the request for examination. The fee to be paid is 50% of the examination fee and one further processing fee of EUR 255 for missing the request for examination. See Euro-PCT Guide, 12th edn., 1 January 2019: PCT procedure at the EPO, point 5.19.003. Please note that this question pertains to a Euro-PCT application, and not a Euro-direct application, where EPO Form 1001 is mandatory (Rule 41(1) EPC) and is pre-printed with the request for grant, so that the applicant only has to worry about paying the examination fee with a 50% surcharge (GL A-VI, 2.2). Also, the applicant did not use Form 1200 for entry into the regional phase before EPO, where the request for examination is also pre-printed in item 4.1.

86
Q

An applicant has missed the due date for the payment of the 6th renewal fee under Article 86(1) EPC by exactly 2 months. Will a request for further processing filed today be accepted? Please provide legal basis to support your answer.

A

No. Under Rule 51(2) EPC a grace period of six months is provided for payment of the renewal fee (provided the additional fee is also paid). There is no loss of rights that can be remedied by further processing.

87
Q

The applicant can use further processing to maintain his application, if he fails to pay the renewal fees during the time limit provided for in Rule 51(2) EPC.

A

No. The grace period of Rule 51(2) EPC is excluded from further processing by Rule 135(2) EPC.

88
Q

On 23 June 2019, the applicant receives an Article 94(3) EPC communication from the examining division dated 19 June 2019 and is given a time limit of four months to reply. The applicant does not provide a timely reply, and on 25 November 2019 he receives a
noting of loss of rights pursuant to Rule 112(1) EPC dated 22 November 2019. Is the following statement correct?

On 3 November 2019, the applicant realises that the time limit has expired. He pays the fee for further processing and files a request for extension of the time limit. The applicant will be granted the extension.

A

False. A request for extension of a time limit does not complete the omitted act as required by Rule 135(1) EPC. Requesting an extension is not equivalent to the required act (J 16/92).

89
Q

On 23 June 2019, the applicant receives an Article 94(3) EPC communication from the examining division dated 19 June 2019 and is given a time limit of four months to reply. The applicant does not provide a timely reply, and on 25 November 2019 he receives a
noting of loss of rights pursuant to Rule 112(1) EPC dated 22 November 2019. Is the following statement correct?

On 3 November 2019, the applicant realises that the time limit has expired. He pays the fee for further processing and files a request for extension. He will be sent a notice that the application is deemed to have been withdrawn for failure to respond to the communication from the examining division, and the fee for further processing will be refunded.

A

False. The request for further processing is admissible, as it may be made before receipt of the Rule 112(1) communication (Article 121(1) and Rule 135(1) EPC). The EPO will have to wait until the end of the two-month period of notification of the Rule 112(1) communication (Rule 135(1) second sentence) to decide that the request for further processing is not allowable, because requesting an extension of time does not complete the omitted act (Case Law of the Boards of Appeal, 9th edn., chapter III.D.2, last paragraph, referring to J 16/92). As the further processing request was admissible, though not allowable, the fee for further processing will not be reimbursed.

90
Q

On 23 June 2019, the applicant receives an Article 94(3) EPC communication from the examining division dated 19 June 2019 and is given a time limit of four months to reply. The applicant does not provide a timely reply, and on 25 November 2019 he receives a
noting of loss of rights pursuant to Rule 112(1) EPC dated 22 November 2019. Is the following statement correct?

On 16 November 2019, the applicant responds to the
communication from the examining division and pays the fee for further processing. The request for further processing will be accepted.

A

(c) True. The communication from the examining division is deemed to have been received on 19 June 2019 + 10 days + 4 months = 29 October 2019 (original time limit). No communication has yetbeen received from the
examining division noting loss of rights, but the applicant can still pay the fee for further processing and complete the omitted act (see GL E-VIII, 2).

91
Q

On 23 June 2019, the applicant receives an Article 94(3) EPC communication from the examining division dated 19 June 2019 and is given a time limit of four months to reply. The applicant does not provide a timely reply, and on 25 November 2019 he receives a
noting of loss of rights pursuant to Rule 112(1) EPC dated 22 November 2019. Is the following statement correct?

On 30 November 2019, the applicant reacts to the noting of loss of rights communication by filing a response to the communication from the examining division and paying the fee for further processing. The request for further processing will be accepted and the legal consequences of his failure to observe the time limit will be deemed not to have ensued.

A

True. The applicant requested further processing by paying the fee within two months of the
communication concerning failure to observe the time limit/loss of rights and responded to the examining division’s communication within the two-month period
(Rule 135(1) EPC).

92
Q

An applicant fails to enter the European regional phase under the PCT within 31 months as specified in Rule 159(1) EPC. He requests further processing within two months. The international application is in French. What is the fee for further processing?

(a) EUR 265 as prescribed in Article 2(1)(12) RFees.
(b) 50% of the fees due upon entry into the European regional phase.
(c) Fees due upon entry into the European regional phase + 50% of the missed fees.
(d) EUR 265 + 50% of the fees due upon entry into the European regional phase, as prescribed in Article 2(1)(12) RFees.

A

(a) False. EUR 265 relates to the missed procedural acts, especially requesting examination. In addition, there is a second further processing fee of 50% of the fees due upon entry into the European regional phase (see box II of the table in GL E-VIII, 3.1.3).

(b) False. EUR 265 must be paid for the missed
procedural acts, especially requesting examination, in addition to 50% of the fees missed.

(c) False. This is not the “further processing fee”, but the total fee that the applicant has to pay to enter the regional phase in Europe. Also, a further processing fee of EUR 265 is missing for failure to request examination.
(d) True. Payment of the missing request for examination (EUR 265) + additionally 50% of the fees due. See Euro-PCT Guide, 12th edn., 1 January 2019: PCT procedures before the EPO, point 5.19.003. Rule 160(1) EPC states that the application is deemed to be withdrawn if the translation of the international application or the request for examination is not filed in due time or if the filing fee, the search fee or the designation fee is not paid in due time (see also box II of the table in GL E-VIII, 3.1.3).

93
Q

Do requests for re-establishment of rights have to be in writing, and is a fee payable? Provide the legal basis to support your answer

A

Yes. It must be in writing and a fee must be paid. The request is not deemed to have been filed until the fee is paid (Rule 136(1) EPC).

94
Q

Can a third party request re-establishment of rights in respect of the time limit for filing an opposition? Provide the legal basis to support your answer.

A

No. The third party is not the applicant or proprietor as required by Article 122(1) EPC.

95
Q

Can an opponent who has filed an appeal request reestablishment of rights in respect of the time limit for filing the grounds of appeal?

A

Yes. An opponent who has filed an appeal can request

re-establishment in respect of the time limit for filing the grounds of appeal (G 1/86; GL E-VIII, 3.1.2).

96
Q

Can re-establishment of rights be granted if the applicant has an accident while on his way to deliver the documents for a European patent application and is therefore unable to meet the priority date to file a European patent application? Why?

A

Yes. Accidents are considered to be exceptional circumstances (T 525/91). The request must be filed within two months of the removal of the cause of compliance, and in particular in relation to the claiming of priority under Article 87(1) EPC, two months from the expiry of the priority period, which is one of the two exceptions to the general possibility of requesting reestablishment within 12 months of the unobserved time limit of Article 122 (see Rule 136(1) EPC).

97
Q

A European patent attorney hired an assistant three months ago. The assistant was new to the world of patents and forgot to input the time limit for requesting further processing of a missed time limit for requesting examination, and did not inform the attorney about the time limit for requesting further processing. Has the “all due care” criterion been demonstrated to an extent that a duly filed request for reestablishment of rights can be granted?

A

No. This is not considered to be an isolated mistake. The assistant had not been sufficiently trained and was not adequately supervised by the patent attorney. Representatives must choose a suitable person for the work, properly instruct them in the tasks to be performed, and exercise reasonable supervision over the work (J 5/80).

98
Q

The assistant of a European patent attorney wrongly calculated the time limit for filing grounds of appeal. The attorney did not have time to verify this time limit himself and he missed the deadline for filing the grounds of appeal. Has the “all due care” criterion been demonstrated to an extent that a duly filed request for reestablishment of rights can be granted?

A

No. The attorney did not act in accordance with his responsibilities (see the discussion under section 3.2.7 Assistants to professional representatives). Where properly selected and instructed, assistants may be entrusted with routine tasks, including noting time
limits. However, particularly urgent time limits which require particular attention and where an irrevocable loss of rights can occur if any error or delay happens are examples of situations which are not routine, and where the representative needs to check the time limit.

99
Q

A European patent attorney misses the scheduled oral
proceedings before the examining division. The result of the oral proceedings conducted in his absence was refusal of the application. Can re-establishment of rights be granted in such a case?

A

No. A specific date – not a time limit – is set for oral proceedings, so missing this date cannot be remedied by means of re-establishment of rights (GL E-VIII, 3.1.1, third paragraph). There is no need to request re establishment of rights, as the decision of the examining division can be appealed.

100
Q

The priority year for a particular application ended on 26 June 2019, and was missed by the applicant despite exercising all due care. The removal of the cause for non-compliance was on 1 August 2019. The request for re-establishment of rights was duly filed on 11 September 2019. Will re-establishment of rights be granted?

A

No. The time limit for filing the request is two months from expiry of the priority year. The request must be filed within two months of the removal of the cause of compliance, and in particular in relation to the claiming of priority under Article 87(1) EPC, two months from the expiry of the priority period, which is one of the two exceptions to the general possibility of requesting reestablishment within 12 months of the unobserved time limit of Article 122 (see Rule 136(1) EPC).

101
Q

Does the EPO grant requests for re-establishment of the right to file a request for re-establishment? Provide the legal basis to support your answer.

A

No. See Article 122(4) EPC and also Rule 136(3) EPC.

102
Q

An applicant was unable to meet a time limit for filing an appeal despite taking due care to monitor the time limit. The time limit for filing the appeal expired on 6 March 2019. The cause for non-compliance was removed on 21 February 2020. Today is 6 March 2020.
Which of the following statements are true and which are false?

(a) The applicant should request re-establishment of rights by filing a written request including the facts of the case and the grounds for re-establishment and paying the fee for re-establishment of rights. It is sufficient to do this by 20 April 2020.
(b) It is sufficient for the applicant to file a written statement of intent to request re-establishment of rights and pay the fee today. The statement of facts and grounds can still be filed next week.
(c) The applicant should request re-establishment of rights by filing a written request, including the facts of the case and the grounds for re-establishment, and paying the fee for re-establishment of rights. All these acts must be done today.
(d) Together with the request for re-establishment of rights, the applicant must also provide evidence to back up the facts on which he relies for his case.

A

(a) False. The time limits of Rule 136(1) EPC must both be met. The one-year time limit of Rule 136(1) EPC expires today, so any request for re-establishment of rights received after today is necessarily inadmissible.
(b) True. The boards of appeal held in J 6/90 that if the two-month period of Rule 136(1) EPC expires later, it is sufficient to file a clear, documented statement of intent to request re-establishment within the one-year period. The grounds must then follow within the two-month period.
(c) False. In view of J 6/90 it is not necessary to file the statements of grounds today.
(d) False. Although Rule 136(2) EPC requires a statement of the grounds on which the request is based and the facts on which it relies to be filed within the time limit, it does not require any evidence backing up those facts. Such evidence can be provided at a later date if necessary (T 324/90; T 667/92; T 261/07).

103
Q

Which of the following statements are true and which are false?

(a) Re-establishment is available to an applicant who failed to file a divisional application while the parent application was pending.
(b) Re-establishment is available to an applicant where a national patent office fails to forward an application to the EPO.
(c) Re-establishment is available to an opponent who misses the time limit for filing an appeal.
(d) Re-establishment is available if the time limit for requesting further processing is missed.

A

(a) False. The requirement that a parent application should still be pending for a divisional application to be filed (Rule 36(1) EPC) is a condition, not a time limit, and is not subject to re-establishment of rights (e.g. J 21/96; J 10/01).

(b) False. Article 122(1) EPC limits re-establishment of rights to “an applicant … or proprietor … who …was unable to observe a time limit vis-à-vis the European Patent Office”. Hence, if a national patent office fails to forward an application to the EPO, it is not possible for the applicant to be granted re-establishment of rights,
since it was not the applicant who had to observe the time limit (J 3/80).

(c) False. An opponent cannot have his rights re-established if he misses the time limit for filing an appeal (T 210/89; GL E-VIII, 3.1.2). However, re-establishment of rights has been extended to allow an opponent who has filed an appeal to request
reestablishment in respect of the time limit for filing the grounds of appeal (G 1/86; GL E-VIII, 3.1.2)

(d) True. Re-establishment of rights is available in respect of the time limit for requesting further processing (GL E-VIII, 3.1.1, second paragraph).

104
Q

A European patent application has an earliest priority date of 1 June 2016 and is directed to substance Z. The European search report mentions a PCT application which has an earliest priority date of 1 March 2016 and a publication date in September 2017
and which clearly discloses substance Z. Does the European patent application lack novelty?

a) Yes, provided the European regional phase is entered.
b) Yes, the PCT application will always destroy the novelty of the European patent application.
c) Yes, provided the European regional phase is entered, but only for commonly designated states.
d) No.

A

a) is correct. For a PCT application to be novelty- destroying under Article 54(3) EPC the European regional phase must be entered. Under EPC2000, the PCT application is relevant to all EPC contracting states.

105
Q

An independent claim concerns a lanthanum salt for removal of dental plaque, and therefore inhibit tooth decay. The prior art shows the use of salts of elements, including lanthanum, to reduce the solubility of tooth enamel in organic acids, and therefore to inhibit tooth decay. Is the invention as claimed novel?

A

Yes. The removal of dental plaque is a method of treatment of the human or animal body under Art. 53(c), and a claim directed to the use of a composition for such removal therefore falls under the exceptions of Art. 54. In this case, since there is already a known medical use of the lanthanum salt, Art. 54(5) is relevant. The removal of plaque is a new
use of the composition compared to the reduction of the solubility of tooth enamel and is therefore novel.

106
Q

An earlier disclosure caused by evident abuse is not novelty destroying if (choose one):
a) It occurred no earlier than six months preceding the actual
filing date of the European patent application.
b) It occurred no earlier than six months preceding the priority
date of the European patent application.
c) It occurred at any time before the actual filing date of the
European patent application.

A

a) is correct. The relevant date is the actual filing date of the European patent application. An earlier disclosure is not taken into account if it occurred no earlier than six months preceding the actual filing date of the European patent application.

107
Q

Applicant X exhibited a new product at an exhibition in Munich four months ago. Do you expect the applicant to be able to take advantage of the provisions of Article 55(1)(b) EPC?

a) No, there is no indication that the product was exhibited at an official, or officially recognised, international exhibition.
b) No, the applicant does not appear to have satisfied the requirements of Article 55(2) and Rule 25 EPC.
c) Yes, the exhibition was clearly an official, or officially recognised, international exhibition as required by Article 55(1)(b) EPC.

A

a) is correct. The provisions of Article 55(2) EPC only apply to a small number of exhibitions, which are listed in the Official Journal. There is no reason to assume that the exhibition in Munich was one of them. If the exhibition had been on the list, the requirements of Article 55(2) and Rule 25 EPC would only have to be satisfied at the time of filing.

108
Q

A stainless steel containing the transition element vanadium or
chromium is known. The European patent application claims a stainless steel containing any transition element. Is the claim novel?

A

No. The specific examples of vanadium and chromium take away the novelty of the generic requirement for any transition element.

109
Q

A patent claims a shampoo comprising 5-25% by weight anionic surfactant and 0.001-0.01% by weight cationic polymer. The prior art describes a shampoo comprising 5-25% by weight anionic surfactant and 0.001-5% by weight cationic polymer. Is the selected range likely to be novel?

a) No, the end points of the selected sub-range are not sufficiently far removed from the end points of the known range.
b) No, the selected sub-range is not narrow compared with the known range and its end points are not sufficiently far removed from the end points of the known range.
c) Yes, the claimed range meets the requirements for a selection invention.

A

a) is correct. The sub-range is less than 1% of the original range and is therefore likely to be considered to be narrow. However, the lower ends of the ranges for the cationic polymer
coincide and there is no suggestion of a purposive selection.

110
Q

The subject-matter of a claimed invention formed part of a book written by the inventor-applicant and published after the priority date. However, shortly before the priority date the inventor-applicant gave the book publisher permission to disclose the content of the book. Was the subject-matter made available to the public before the priority date?

A

No. It cannot be assumed that permission to make the invention available to the public means that this actually happened.

111
Q

A patent is directed to the invention of substance X and has been opposed on the basis of a product information sheet from a third party bearing a printing date eight months before the priority date of the patent. No evidence is presented that the product information sheet was actually distributed to a member of the public before the priority date. Is it likely that the product information sheet will be considered to have been made available to the public before the priority date?

A

Yes. The test to be applied is the balance of probabilities, and on the basis of this test the document can be expected to have been made available to the public within the relatively long period of eight months.

112
Q

A claim is directed to a tile which includes a lighting element and energy supply means in the form of electrical wiring. The prior art discloses a tile including a lighting element in which the energy supply means was in the form of optical fibres. Does the prior art destroy the novelty of the claim?

A

No. Optical fibres are not electrical wiring; they are only equivalent to electrical wiring. They are not the same because electrical wiring carries electrical energy, while optical fibres carry energy in the form of light. The claim is therefore novel.

113
Q

Claim 1 of a European patent is directed to a disposable absorbent garment (nappy or diaper) having an impervious backing and a permeable body side lining with an absorbent material between the backing and the lining, characterised in that from 55 to 75 percent of the total amount of absorbency of the garment is in the front half of the garment.

The prior art shows a similar garment. The opposition division revoked the patent for lack of novelty based on a calculation of absorbency based on the drawings of the prior art, although there was no mention in the prior art of any range of values of absorbency with respect to either half of the garment. The patent proprietor appealed. Was the appeal successful in respect of novelty?

A

Yes. The board of appeal concluded that the assumptions made for the calculation went beyond what could be directly and unambiguously derived from the prior art and could not be used to deprive claim 1 of novelty.

114
Q

Consider the following claim:
“A container having a cover, the container comprising a hinge to move the cover relative to the container and/or a handle.”

You become aware of prior art disclosing a container having a cover, the container comprising a hinge to move the cover relative to the container.

Is the subject-matter of the claim novel over the prior art?

A

No. The prior art discloses one of the alternatives covered by the claim, so the claim as a whole lacks novelty.

115
Q

The term “combination invention” applies to an invention in which (choose one or more):

(a) The interactions of the individual features of a claim produce a synergistic effect.

(b) The functional interaction between the claim features achieves a combined technical effect which is greater than the sum of the technical effects of the individual features.
(c) There is a combination of features which are cumulatively
aggregated to produce the claim.

A

a) and (b) are correct. Another way of
explaining this would be to say that the interactions of the individual features of a claim produce a synergistic effect. A synergistic effect exists if the functional interaction between the claim features achieves a combined technical effect which is greater than the sum of the technical effects of the individual features. A combination
invention is an invention in which the interactions of the individual features of a claim produce a synergistic effect, not merely an aggregation. Another way of putting this is to say that the functional interaction between the claim features achieves a combined technical effect which is greater than the sum of the technical effects of the individual features.

116
Q

In the case of a claim which presents a number of “partial problems”, there is an inventive step:

(a) If the solution to any one of the partial problems is not obvious.
(b) Only if the solutions to all of the partial problems are not obvious.

A

(a) is correct. There is an inventive step if the solution to any one of the partial problems is not obvious.

117
Q

True or false? A first request by a party for oral proceedings
may not be denied:
(a) By the Receiving Section.
(b) By an examining division.
(c) By an opposition division.
(d) By a board of appeal.
(e) Two days prior to the date stamped on a decision by the
examining division in respect of an application.

A

(a) False. A request for oral proceedings may be denied if the Receiving Section considers that they are not expedient or does not intend to refuse the application (Article 116 EPC).
(b) True. Oral proceedings take place in accordance with Article 116 EPC, provided that the subject-matter of the proceedings has not been dealt with in previous oral proceedings.
(c) True. See the answer to question (B).
(d) True. See the answer to question (B).
(e) False. Oral proceedings can only be requested prior to the termination of proceedings. Requests for oral proceedings must be allowed as long as proceedings before the EPO are still pending, i.e. until the decision to grant has been handed over to
the internal postal service (see G 12/91 and T 556/95, especially point 4.4 of the Reasons; GL E-III, 2; GL C-V, 4.7.1). According to G 12/91, the date of dispatch used to be stamped on
the decision three days before it was actually dispatched (G 12/91, point 9.1 of the Reasons). So, presenting a request for oral proceedings two days prior to the date stamped on a decision in the relevant case is too late because at that point in time, it has already been handed over to the EPO postal service for notification. However, it seems that this three-day rule for handing over to the postal service is no longer strictly adhered to at the
EPO. It can therefore no longer be guaranteed that even a request submitted four days before the date stamped on the decision will always be on time.

118
Q

True or false? When a summons to attend oral proceedings is
notified to a party:
(a) The parties will be given at least three months’ notice.
(b) Notification must be sent by registered post with advice of
delivery.
(c) The parties will always be consulted about the date of the
proceedings.
(d) The proceedings may be set to last more than one day.
(e) A request to postpone oral proceedings before the opposition division will not be allowed if the request is due to the fact that the representative is too busy or has scheduled a business trip on the appointed date.

A

(a) False. The parties are given at least two months’ notice of the summons, i.e. the notice given in the summons to oral proceedings must be at least two months, unless they agree to a shorter period (Rule 115(1) EPC).Normally, the summons is issued at least four months ahead of the day of the oral
proceedings in examination and at least six months ahead of the day of the oral proceedings in opposition (GL E-III, 6).
(b) False. Summons (e.g. to oral proceedings) are notified by registered letter but no longer with advice of delivery (Rule 126(1) EPC).This change is effective as of 1 November 2019, see the notice from the EPO in OJ EPO 2019, A57 and Enlarged Board of Appeal G 1/14 (Annex I).
(c) False. In first-instance ex parte proceedings, the examining division is allowed to contact the applicant when fixing the date (see the notice from the EPO in OJ 2009, 68), but it does not have to, while the boards of appeal do not consult the parties.
(d) True. In complex cases before the divisions or boards of appeal, oral proceedings may be set to last more than one day (GL E-III, 6).
(e) True. The opposition divisions take a stricter approach than the examining divisions (T 1102/03; GL E-III, 7.1.1).

119
Q

True or false?

(a) The public may attend all oral proceedings before the boards of appeal.
(b) The use of computer-generated slide presentations during oral proceedings is prohibited.
(c) A representative who is required to file an authorisation for a party may be allowed to supply their authorisation after the oral proceedings, within a time limit of three months.
(d) Even if a party arrives late, as long as the oral proceedings have not ended, it has a right to be heard.
(e) Oral proceedings will always be cancelled if a party to the opposition proceedings announces that it will not attend them.

A

(a) False. Although oral proceedings are public, the public may not attend proceedings before the boards of appeal (or the Enlarged
Board of Appeal or the opposition divisions) if their admission could have serious and unjustified disadvantages, in particular for a party, or where proceedings concern an application prior to its publication (Article 116(4) EPC).
(b) False. Computer-generated slide presentations are not a matter of right, but may be allowed if permission is asked of the relevant division (T 1556/06; GL E-III, 8.5.1).
(c) False. The time limit set for a representative to provide written authorisation is two months (GL E-III, 8.3.1).
(d) True. A party to oral proceedings, even if arriving late, is entitled to be heard (GL E-III, 8.3.3.1).
(e) False. If a party has indicated that it will not attend, it is within the discretion of the division to decide whether the scheduled oral
proceedings are to be maintained or cancelled (GL E-III, 7.2.2).

120
Q

True or false? During oral proceedings:

(a) An expert for a party may give their opinion on all aspects of the proceedings.
(b) A party to the proceedings, which has its residence or principal place of business in a contracting state, always has the right to speak, in addition to its professional representative
(c) An expert is always allowed to speak, provided that a request is filed in due time by a party to the proceedings.
(d) If the EPO allows a party to make a visual presentation, the EPO will provide a projector and computer facilities.

A

(a) False. Experts are limited to the subject-matter outlined in the request to give evidence during oral proceedings (GL E-III, 8.5).
(b) True. By virtue of being a party to the proceedings, the party has the right to be heard during the oral proceedings (GL E-III, 8.5; G 4/95; T 621/98, Article 133(2) EPC).A party to the proceedings that has its residence or principal place of business in a
contracting state always has the right to speak, in addition to its professional representative.
(c) False. An accompanying expert is allowed to speak only at the discretion of the examining or opposition division (GL E-III, 8.5; G 4/95).
(d) False. Requests to the EPO to provide computer equipment for slide presentations must be denied, but the EPO will provide a screen (GL E-III, 8.5.1).

121
Q

True or False
a) The time limit for filing written submissions prior to oral
proceedings is fixed in Rule 115 EPC.
(b) Late-filed facts and evidence are treated more seriously when a party is absent from the oral proceedings.
(c) A party’s right to be heard is an important consideration in the conduct of the oral proceedings.
(d) Only the chairperson, and not the other members of the division, is permitted to speak during oral proceedings.

A

(a) False. Although the deadline for filing written submissions prior to oral proceedings is given in the notification of the summons to oral proceedings (Rule 115(1) EPC), the final date is fixed in accordance with Rule 116(1) EPC. See GL E-III, 8.6; GL E-VI, 2.
(b) True. A party may not be taken by surprise by the submission of facts or evidence that it had no grounds for expecting (Rule 115(1) EPC; GL E-III, 8.3.3.2).
(c) True. Each party has a right to comment on facts, evidence and arguments put forward by the other parties (Article 113(1) EPC; GL E-III, 8.6; T 117/02).
(d) False. The chairperson of the examining or opposition division must allow the other members to ask questions at their request, but may decide at which point in the
proceedings this should take place (GL E-III, 8.10).

122
Q

True or false?
(a) Oral proceedings can be held in an official language of any
contracting state, and the EPO will arrange for interpretation provided that the party gives one month’s notice.
(b) When the EPO arranges for interpretation at oral proceedings, a party may listen to the proceedings in one language and answer in another. (You may wish to read T 774/05, especially point 6 of the Reasons, before answering this question.)
(c) Amended claims submitted during oral proceedings must be in the language of the proceedings.
(d) The parties may request correction of the minutes of the oral proceedings if they have reason to believe that a party’s statement has been omitted or incorrectly recorded.

A

(a) False. While a party may use any official language of a contracting state, it is responsible for making provision for interpretation into the language of the proceedings (Rule 4(1) and (4) EPC; GL E-V, 2).
(b) False. A party does not have the right to speak in one language and hear the proceedings in another (Rule 4(1) and (5) EPC;
GL E-V, 1; T 774/05).
(c) True. Amendments to the application submitted during oral proceedings must be in the language of the proceedings (Article 14(3) EPC; Rule 3(2) EPC).
(d) True. If a party finds that the minutes do not fulfil the requirements of Rule 124(1) EPC, it has the right to request correction and propose an alternative text (GL E-III, 10.4).

123
Q

A notice of opposition is filed with the EPO within the time limit of Article 99(1) EPC in one of the official languages. The notice is duly signed and sufficiently identifies the patent to be opposed. EPO Form 2300 E is not used and the opposition fee is received at the EPO three days after expiry of the nine month time limit of Article 99(1) EPC.

Which of the following statements is true?

(a) The formalities officer will issue a communication to the opponent and invite them to supply EPO Form 2300 E within a time limit. If the opponent fails to do so, the opposition will be deemed not to have been filed.
(b) The formalities officer will issue a communication to the opponent and invite them to supply EPO Form 2300 E within a time limit. If the opponent fails to do so, the opposition will be rejected as inadmissible.
(c) Owing to the late payment of the opposition fee, the opposition will be rejected as inadmissible.
(d) Owing to the late payment of the opposition fee, the opposition will be deemed not to have been filed, and the opposition fee will be refunded.

A

(a) False. The use of Form 2300 E to file a notice of opposition is not mandatory. So, no formal error is invoked by not using the form. In contrast, when filing a European patent application with the EPO, the use of EPO Form
1001 is compulsory (GL A-III, 4.1).
(b) False. See the answer to(a).
(c) False. According to Article 99(1), second sentence, EPC, notice of opposition is not deemed to have been filed until the opposition
fee has been paid (GL D-IV, 1.2.1 (i)).
(d) True. See the answer to question (c). Since the opposition has not started, the opposition fee will be refunded (GL D-IV, 1.4.1)

124
Q

A notice of opposition is filed with the EPO two weeks before
expiry of the nine-month time limit of Article 99(1) EPC. However, the notice is in an official language of a contracting state which is not an official language of the EPO. The notice complies with the formal requirements. The opposition fee is paid on the same day. Two weeks after expiry of the nine-month time limit of Article 99(1) EPC, the EPO receives a translation of the notice of opposition into an official EPO language.

Which of the following statements is true?

(a) According to Article 14(4) and Rule 3(1) EPC, notice of opposition can be filed in an official language of a contracting state which is not an official EPO language. A translation into an official EPO language must however for reasons of legal certainty be filed within the nine-month time limit of Article 99(1) EPC. In the present case, the notice of opposition is therefore deemed not to have been filed.
(b) According to Article 14(4) and Rule 3(1) EPC, a notice of opposition can be filed in an official language of a contracting state which is not an official EPO language. A translation can still be filed within one month of filing of the notice of opposition in the official language of the contracting state, even if the nine-month time limit has expired.
(c) The provisions of Article 14 and Rule 3 EPC regarding languages apply to patent applications and other submissions filed in examination proceedings. They are not applicable to opposition proceedings.
(d) According to Article 14(4) and Rule 3(1) EPC, a notice of opposition can be filed in an official language of a contracting state which is not an official EPO language. Failure to supply a translation within the nine-month time limit of Article 99(1) EPC is a formal error which will result in the formalities officer issuing a communication setting a time limit within which the translation has to be filed.

A

(a) False. According to Article 14(4) EPC and Rule 3(1) EPC, a notice of opposition can be filed in an official language of a contracting state which is not an official EPO language. A translation into an official EPO language must be filed within the one-month period as required under Rule 6(2) EPC, but this period is extended to the end of the nine-month time limit of Article 99(1) EPC for filing the opposition where this time limit expires later (GL D-IV, 1.2.1 (v), referring to G 6/91; GL A-VII, 3.2).
(b) True. See the answer to question (a).

(c) False. The language regime of Article 14 and Rule 3 EPC applies to patent applications and other submissions filed in examination proceedings as well as to the notice of
opposition and other submissions filed in opposition proceedings. It also applies to appeal proceedings (G 6/91).

(d) False. Failure to supply the translation within the nine-month time limit of Article 99(1) EPC will not result in the formalities officer issuing a communication with a time limit, as the time limits under Article 99(1) EPC and Rule 6(2) EPC are not Office time limits.

125
Q

A notice of opposition is filed with the EPO within the time limit of Article 99(1) EPC. The notice fulfils all formal requirements except for the fact that the proprietor of the patent has not been indicated. The opposed patent can, however, be identified by the EPO. This deficiency is not remedied within the time limit set by the formalities officer, who after expiry of this time limit notifies the opponent that the opposition is likely to be rejected. In a further communication, the formalities officer rejects the opposition as inadmissible. Six weeks later it turns out that the opponent had been unable to reply to the formalities officer’s communication because of a car accident.

Which of the following statements is true?

(a) The opponent can request a decision under Rule 112(2) EPC.
(b) The opponent can immediately appeal the decision, even though it was issued by the formalities officer rather than the opposition division.
(c) The opponent can request further processing in accordance with Article 121 EPC. Their request will be granted if, within the two month time limit of Rule 135(1) EPC, they also pay the fee for further processing and complete the omitted act, i.e. adequately identify the patent proprietor.
(d) As the opponent was unable to observe the time limit because of the car accident, their rights can be re-established under Article 122 EPC. A car accident is normally an acceptable reason for re-establishment. The opponent must then, in accordance with Rule 136(1) EPC, within two months after their recovery, but at the latest within one year of expiry of the time limit set by the formalities officer, complete the omitted act, pay the fee for reestablishment of rights and state the grounds on which the request is based, as well as setting out the facts (that is, the car accident) on which they rely.

A

a) False. According to Rule 76(2)(b) EPC, the notice of opposition must contain the name of the proprietor of the opposed patent. This applies even when the contested European Patent can be identified without this information (GL D-IV, 1.2.2.2 (ii)). Failure to indicate the name of the proprietor is a deficiency under Rule 77(2) EPC and has to be remedied within the period
stipulated by the formalities officer (GL D-IV, 1.3.2), who will otherwise notify the opponent that the notice of opposition is likely to be rejected as inadmissible (GL D-IV, 1.4.2, first paragraph). Since the name of the proprietor is a deficiency that can be remedied under Rule 77(2) EPC within a time period set, the opponent could still have overcome this objection.
Therefore, a decision will not be taken immediately. However, the opponent failed to meet the deadline.

Since the objection is not overcome and the examiner’s opinion on the failure to name the proprietor was not refuted by the opponent, the formalities officer was competent to reject the notice of opposition as inadmissible (GL D-IV, 1.4.2, first paragraph, referring to the Decision of the President of the EPO dated 12 July 2007 concerning the entrustment to non-examining staff of certain duties normally the responsibility of the examining or opposition divisions, OJ EPO special edition 3/2007, F.2).
Therefore, the formalities examiner has already provided a decision and no further decision can be requested.

(b) True. The formalities officer’s decision is a decision under Article 106(1) EPC.
(c) False. Further processing is limited to the applicant and is therefore not available to either the patent proprietor or the opponent.
(d) False. Re-establishment of rights is limited to the applicant or patent proprietor and is therefore not available to the opponent. An exception applies only in appeal proceedings regarding reestablishment of the right to file a reasoned statement (G 1/86).

126
Q

A notice of opposition in an official EPO language which fulfils the requirements of Rule 76(2)(a), (b) and (d) EPC is filed with the EPO, and the opposition fee is paid within the time limit of Article 99(1) EPC. On the basis of the written submissions contained in the notice of opposition, it is doubtful that the reasoning in the notice can be properly understood on an objective basis. Which of the following statements is true?

(a) The formalities officer will issue a communication stating that the notice of opposition does not comply with the requirements of Rule 76(2)(c) EPC and that the opposition is rejected as inadmissible.
(b) The opposition division will issue a communication informing the opponent that the notice of opposition probably does not comply with the requirements of Rule 76(2)(c) EPC and that the opposition is likely to be rejected as inadmissible unless substantiated reasoning is submitted within a period set by the division.
(c) The opposition division will issue a communication informing the opponent that the notice of opposition probably does not comply with the requirements of Rule 76(2)(c) EPC and will ask the opponent to submit observations. It will decide on the admissibility of the opposition when these observations have been filed.
(d) If the opponent does not refute the opinion of the opposition division regarding the deficiencies, the opposition division will, without the patent proprietor being a party, reject the notice of opposition as inadmissible, possibly after having held oral proceedings.

A

(a) False. In cases of insufficient substantiation, the formalities officer is not competent to decide on the admissibility of the opposition and will forward the notice of opposition to the opposition division (GL D-IV, 3; see also GL D-IV 1.4.2).

(b) False. The opposition division will issue a communication informing the opponent that the notice of opposition probably does not comply with the requirements of Rule 76(2)(c) EPC and that the opposition is likely to be rejected as inadmissible. Insufficient substantiation cannot be remedied by further submissions which will “fill the gap”, as the time limit of Rule 76(2)(c) EPC, in conjunction with Article 99(1) EPC, is an EPC time limit for which no extension is possible. Any time limit set by the opposition division will therefore apply only to submissions of the opponent indicating why the substantiation provided in the notice of opposition should be considered sufficient.
(c) True. In cases where at least some indication of facts and evidence on which the opposition is based was submitted in the notice of opposition, it may not be apparent to the formalities officer that the indication does not fulfil the requirements of Rule 76(2)(c) EPC. The opposition division will issue a
communication informing the opponent that the notice of opposition probably does not comply with the requirements of Rule 76(2)(c) EPC and will request the opponent to submit
observations. It will then decide on the admissibility of the opposition after these observations have been filed (GL D-IV, 1. 3.2; GL D-IV,1.2.2.1 (v)).

(d) True. See GL D-IV, 1.3.2.

127
Q

True or false?
(a) If the parties have been duly summoned and one party does not attend the oral proceedings, the opposition division is free to take any decision during the oral proceedings, as the duly summoned party has deliberately chosen to waive its right to be heard. In addition, the party can still appeal the decision.
(b) Whenever oral proceedings are held, the decision is taken at the proceedings and is announced at the end. Once it has been announced, it stands, i.e. it is final.
(c) When oral proceedings are held and the decision is taken to
maintain the patent in amended form, the decision becomes final as soon as the translations of any amended claims have been filed in the official languages of the EPO which are not the language of the proceedings.
(d) When a decision is taken in written proceedings, any submission which is filed before the opposition division hands the decision over to the internal postal service will be taken into consideration.

A

False. Generally speaking, the non appearance of a party does not prevent the opposition division from taking a legally valid decision, even if the non-attending party is adversely affected by the decision. An exception applies, however, if new facts are taken into consideration and, at the end of the oral proceedings, a decision based on these facts would adversely affect the absent party. In such a case, the absent party could be considered to have been taken by surprise by the course of events, which would be a breach of its right to be heard. Such an exception would apply, for example, if the subject-matter of the contested patent is considered not to extend beyond the content of the application documents as originally filed, but the objection of insufficiency of disclosure is raised for the first time during the oral proceedings (and thus constitutes a fresh ground for opposition) and seems likely to prejudice the patent as granted, or if a novelty-destroying document is filed for the first time during the oral proceedings.

(b) False. Oral proceedings can also end without a decision being taken. The decision can then be taken in written proceedings, or during subsequent oral proceedings. In general, it is not possible to give a decision granting a European patent or maintaining it in amended or limited form in oral proceedings, because further procedures must be followed (GL E-III, 9).
(c) False. In accordance with Rule 82(2) EPC, not only do translations of any amended claims into the other official languages of the EPO have to be filed, but the printing fees have to be paid within a three-month time limit set by the opposition division. As to when the decision becomes “final”, it is worth noting that the decision stands as soon as the requirements of Rule 82(2) EPC are fulfilled, but the time limit for appealing this decision is normally two months (Article 108, first sentence, EPC) plus ten days (Rule 126(2) EPC).
(d) True. Under normal circumstances, this will be the case (GL D-VI, 7.1, referring to G 12/91), although proposals for amendment filed at a late stage in the proceedings may be disregarded (GL D-VI, 4.2, referring to T 406/86).

128
Q

An opposed patent comprises independent claim 1 with dependent claims 2-7 and independent claim 8 (with a clearly different scope) with dependent claims 9-15. Novelty is the sole ground for opposition raised in the notice of opposition. Only independent claim 1 is objected to by the opponent, with only one document forming the state of the art under Article 54(3) EPC. The notice of opposition contains several further documents forming the state of the art under Article 54(2) EPC.
Which of the following statements is correct?
(a) The opposition division is entitled to extend the examination to dependent claims 2-7 if their validity is prima facie in doubt on the basis of the documents cited in the notice of opposition.
(b) The notice of opposition is directed to partial revocation of the patent, i.e. to independent claim 1 only. The opposition division is entitled to extend the examination to dependent claims 2-7 and to independent claim 8 and dependent claims 9-15 if their validity is prima facie in doubt on the basis of the
documents cited in the notice of opposition, as it is the purpose of first-instance proceedings to avoid invalid patents.
(c) With regard to independent claim 1, only novelty is objected to in the notice of opposition as the sole ground for opposition, with a document forming the state of the art under Article 54(3) EPC. At the oral proceedings it turns out that the subject-matter of independent claim 1 is new. The opponent argues that the
subject-matter of claim 1 does not involve an inventive step over some of the other documents cited in the notice of opposition. This objection is permissible.
(d) The novelty of independent claim 1 and dependent claims 2-7 is objected to in the notice of opposition, with a document forming the state of the art under Article 54(2) EPC. The ground of lack of inventive step has also been indicated on EPO Form 2300, but has not been substantiated. At the oral proceedings it turns out that the subject-matter of independent claim 1 is new. The opponent argues that the subject-matter of claim 1 does not involve an inventive step over one of the documents cited in the notice of opposition. This objection is permissible.

A
(a) True. Although the opposition division normally has to limit its examination to the part opposed (GL D-V, 2.1; CLB IV.C.3, referring to G 9/91 and G 10/91), an exception applies where the opposition is directed to an independent claim only and the validity of the dependent claims is prima facie in doubt on 
the basis of the information available (i.e. the Article 54(3) EPC document). 
(b) False. The opposition division is definitely not entitled to extend its examination to independent claim 8, which has a clearly 
different scope (T 525/96).
(c) False (generally speaking). First it should be noted that the ground of inventive step, which was not raised in the notice of opposition, is a fresh ground for opposition (CLB IV.C.3.4, referring to G 1/95 and G 7/95). Furthermore, the documents forming part of the state of the art under Article 54(2) EPC 
 were not substantiated in the notice of opposition, and are therefore considered to be late-filed (GL E-III, 8.6). The objection could be permissible if the documents forming part of the state of the art under Article 54(2) EPC unexpectedly turn out to be prima facie relevant, i.e. if they appear to affect the outcome of the proceedings (T 320/15).
(d) True. Under these circumstances inventive step can be considered as having been raised as a ground for opposition because its substantiation would have contradicted the opponent’s reasoning with regard to lack of novelty (CLB IV.C.3.4.2, referring inter alia to T 131/01 and T 635/06)
129
Q

True or False
(a) An assumed infringer can intervene in opposition
proceedings if proceedings for infringement of the European patent have been instituted against them or if they have instituted proceedings for a ruling that they are not infringing the patent.
(b) An assumed infringer can intervene in opposition proceedings if proceedings for infringement of the European patent have been instituted against them or if, following a request of the proprietor to cease alleged infringement, they have instituted proceedings for a ruling that they are not infringing the patent, or if the proprietor has launched infringement proceedings against a parallel national patent.
(c) An assumed infringer who intervenes in opposition proceedings acquires the status of an opponent and has to pay an opposition fee.
(d) An assumed infringer who has not intervened during opposition proceedings can still do so if the two-month time limit of Article 108, first sentence, EPC has not yet expired. They then have to pay an opposition fee and they acquire the legal status of an opponent.

A

(a) False. An assumed infringer can intervene in opposition proceedings if proceedings for infringement of the European patent have been instituted against them or where they have instituted proceedings for a ruling that they are not infringing the patent if the institution of these proceedings following a request by the proprietor to cease the alleged infringement (Article 105(1)(b) EPC).
(b) False. An assumed infringer can intervene in opposition proceedings if proceedings for infringement of the European patent have been instituted against them or where,
following a request by the proprietor to cease the alleged infringement, they have instituted proceedings for a ruling that they are not infringing the patent (Article 105(1)(a) and (b) EPC). However, infringement proceedings in respect of a national patent do not entitle the assumed infringer to intervene in opposition proceedings.
(c) True. An assumed infringer who intervenes in opposition proceedings acquires the status of an opponent and has to pay an opposition fee (G 3/04).
(d) False. An assumed infringer who has not intervened during opposition proceedings cannot necessarily appeal. They can still do so if the decision of the opposition division is appealed by at least one opponent, and in this case even after the two-month time limit of Article 108, first sentence, EPC has expired, because the appeal is pending. In this case, they have to pay an opposition fee and they acquire the legal status of an opponent. If the
opposition proceedings are no longer pending, the proceedings cannot be continued with the assumed infringer only (G 4/91).

130
Q

(a) A request for an order for apportionment of costs can be made only for expenses necessary to ensure proper protection of the rights involved. The costs for remuneration of the professional representative are part of these expenses, but additional expenses incurred by an employee cannot be taken into consideration.
(b) If a party agrees in principle with the order for apportionment of costs, but is of the opinion that the costs put forward by the other party are at least partly unjustified, the appropriate legal remedy is to lodge an appeal against the final decision and to state in this appeal the grounds why the order for apportionment of costs is considered inappropriate.
(c) If a party disagrees with the order for apportionment of costs, the appropriate legal remedy is to appeal the order for apportionment of costs.
(d) After the opposition division has taken a final decision on the substantive matters of the contested patent, a request for an order for apportionment of costs can be made to the formalities officer, who will fix the costs and notify the parties accordingly.

A

(a) False. A request for an order for apportionment of costs can indeed be made only for expenses necessary to ensure proper protection of the rights involved (Rule 88(1), first sentence, EPC). Costs for remuneration of the professional representative are part of these expenses (Rule 88(1), second sentence, EPC). Additional expenses incurred by an employee can be taken into consideration if they were necessary to ensure proper protection (CLB III.R.3, referring to T 930/92; see also GL D-IX, 1.3).
(b) False. In this case the most appropriate measure would be to file a request under Rule 88(3) EPC for a decision by the opposition division and to state the grounds why the respective costs are considered not to be credible.
(c) True (in principle). Note, however, that according to Article 106(3) and Rule 97(1) EPC the apportionment of costs
cannot be the sole subject of an appeal. In order to validly challenge the order for apportionment, the entire decision of the opposition division must be appealed.
(d) False. A request for an order for apportionment must be made before any decision is announced in oral proceedings (CLB III.R.4.1, referring to T 212/88). Likewise, it can be assumed that in written proceedings the request must be filed before any decision is taken. However, even when the opposition division has ordered an apportionment of costs, the formalities officer will fix the amounts to be paid only at the request of a party (Rule 88(2), first sentence, EPC).Thus, the party must still take this extra action after the final decision in the case, regardless of the order for apportionment by the opposition division.

131
Q

Can an appeal be filed against any EPO communication?

A

No. An appeal can only be filed against a “decision” of the EPO (Article 106(1) EPC).

132
Q

Can an appeal be filed by any party to the proceedings?

A

No. An appeal can only be filed by a party to the proceedings who is adversely affected by a decision (Article 107 EPC).

133
Q

Can an alleged infringer intervene in opposition appeal

proceedings?

A

Yes. An alleged infringer may also intervene in pending appeal proceedings following opposition (G 1/94).

134
Q

Does an alleged infringer intervening in opposition appeal proceedings have to pay the appeal fee?

A

No. An alleged infringer needs to file notice of opposition and pay an opposition fee (G 1/94).

135
Q

n opposition proceedings, the patent proprietor has successfully defended their patent and had it maintained in amended form based on their first auxiliary request, but they have not withdrawn their main request. The opponent had requested that the patent be revoked. Who can appeal such a decision?
A. The proprietor can appeal.
B. The opponent can appeal.

A

Both A and B are correct. Both parties are adversely affected (Article 107 EPC) and can appeal. The proprietor was not granted their main request (as the patent was upheld in amended form) and the opponent was not granted their request for revocation of the patent.

136
Q

Following on from question 5, the opponent files an appeal requesting revocation and the proprietor becomes party to the proceedings as of right (Article 107 EPC). Is it possible for the proprietor as a party as of right to request that the patent be maintained on the basis of their broader main request?

A

No. If the proprietor does not appeal, they may only defend the patent as maintained in opposition proceedings, i.e. on the basis of the first auxiliary request. This is because they did not appeal and because the opponent/appellant must not be put in a worse position after appeal than if they had not appealed (reformatio in peius, G 4/93 – see also section 5.3.3)

137
Q

Can appeals be filed online?

A

Yes. Appeals may be filed by any means of communication
permitted under Rule 2(1) EPC. This includes electronic filing, but not Web-Form Filing (Decision of the President of the EPO, OJ EPO 2018, A45)

138
Q

If the sole appellant withdraws their appeal, can the board continue to examine the facts of its own motion?

A

No. While opposition proceedings can be continued ex officio
under Article 114(1) EPC, that article does not sanction the continuation of appeal proceedings after the appeal has been
withdrawn (G 7/91; G 8/91).

139
Q

Where a new document is raised in appeal proceedings, does the
applicant have the right to have the case remitted to the department of first instance (Article 111 EPC)?

A

No. Under Article 111(1) EPC, the board may either decide on the appeal or remit the case to the department responsible for the appealed decision to deal with outstanding issues. The decision on whether or not to remit a case is at the board’s discretion; there is no right to remittal. Under the RPBA 2020, the board will only remit cases to the department whose decision was appealed for further prosecution if special reasons present themselves for doing so

140
Q

From the date of entry into force of EPC2000, can any party adversely affected by a decision of a board of appeal file a petition for review by the Enlarged Board of Appeal?

A

Yes. However, this is a limited judicial review by the Enlarged Board of Appeal and is not a review of the application of substantive law (R 2/08; R 9/08; R 13/09). The petition for review is not a comprehensive review of aboard of appeal decision. It provides a remedy in individual cases where fundamental procedural defects have occurred (Article 112a(1) EPC).

141
Q

Do petitions for review have suspensive effect in accordance

with Article 112a(3) EPC?

A

No. Petitions for review do not have suspensive effect in
accordance with Article 112a(3) EPC, since they are outside the
normal framework of the instances of the EPO. Therefore, the board of appeal’s decision has legal effect, unless and until an Enlarged Board decision sets it aside and re-opens appeal proceedings.

142
Q

Is there a time limit for filing requests for revocation?

A

No. Article 105b and Rules 90 to 96 EPC do not specify any time limit, so there is no restriction on the period between the grant of the patent and the filing of the request. Accordingly, requests can be filed at any time after grant or after opposition proceedings, or even after the patent has expired (GL D-X, 1).

143
Q

Is there a time limit for filing requests for limitation?

A

No. Article 105b and Rules 90 to 96 EPC do not specify any time limit, so there is no restriction on the period between the grant of the patent and the filing of the request. Accordingly, requests can be filed at any time after grant or after opposition proceedings, or even after the patent has expired (GL D-X, 1).

144
Q

Is it necessary to give reasons for a request for revocation?

A

No. According to Article 105b and Rules 90 to 96 EPC, this is not necessary.

145
Q

Is it possible to file another request for limitation if a first request
is rejected?

A

Yes. The EPC does not place a limit on the number of successive requests for limitation that can be filed.

146
Q

Can a third party file observations under Article 115 EPC while
limitation proceedings are pending?

A

Yes. Article 115 EPC covers all proceedings before the EPO. The patentability referred to in Article 115 EPC extends to Article 84 and Article 123(2) EPC. During limitation proceedings the patent is examined for compliance with both Article 84 and Article 123(2) EPC (Rule 95(2) EPC; GL D-X, 4.5)

147
Q

Is it possible to appeal the examining division’s decision not to
grant a request for limitation?

A

Yes. The rejection of a request for limitation can be appealed (GL D-X, 6).

148
Q

When deciding on the allowability of a petition for review, does
the Enlarged Board always comprise five members?

A

No. In the first step of the two-step procedure, the Enlarged Board of Appeal comprises three members (Rule 109(2)(a) EPC). If they unanimously consider that a request is not allowable, the request will be rejected. Otherwise, there will be a second review. In this second review step, the Board has five members (Rule 109(2)(b) EPC).

149
Q

Will the other parties to the proceedings that led to the impugned decision of the board of appeal always be heard by the Enlarged Board of Appeal on the allowability of a petition for review?

A

No. They will not always be heard. In its three-member composition the Enlarged Board will decide without the involvement of other parties on whether or not the petition is clearly inadmissible or unallowable (Rule 109(3) EPC). However, during the second step (Rule 109(2)(b) EPC), other parties to the proceedings will have the opportunity to be heard.

150
Q

The PCT currently (April 2020) has 153 contracting states, but
how many international authorities can act as ISA, and where can you find this information?

A

Currently (April 2020), a total of 23 international authorities act as ISA/IPEA. The information can be found in AG-IP, Annex D. Also see http://www.wipo.int/pct/en/access/isa_ipea_agreements.html.

151
Q

An international application is filed with the Danish Patent and
Trademark Office as receiving Office. Which of the following ISAs
is/are competent?
For each of statements A to F, indicate whether the statement is true or false.
A. The Danish Patent and Trademark Office (DK).
B. The Swedish Patent and Registration Office (SE).
C. The Nordic Patent Institute (XN).
D. The European Patent Office (EP).
E. It depends on the nationality and place of residence of the
inventors and applicants.
F. The International Bureau (IB).

A
A. False.
B. True.
C. True.
D. True.
E. False.
F. False.
The competent ISAs are: the European Patent Office (EP), the Nordic Patent Institute (XN) and the Swedish Patent and Registration Office (SE) (AG-IP, Annex C, DK). The competent receiving Offices are determined by the nationality and residence of the inventors/applicants (Rule 19.1 PCT). The competent ISAs are determined by the agreement made between the receiving Office and WIPO under Article 16(2) PCT.
152
Q

An applicant from the USA intends to file an international
application relating to business methods with the USPTO as receiving Office, and is considering selecting the EPO as ISA. Is it possible for him to do so, and, if so, would it be a good idea?

A

Yes. A US applicant can select the EPO to act as ISA (AG-IP, Annex C, US), but it would probably not be a good idea.
Following an amendment to the Agreement between the European Patent Organisation and the IB of WIPO, any national or resident of the USA filing an international application with the United States Patent and Trademark Office (USPTO) or the IB as receiving Office can select the EPO as ISA irrespective of the technical field in which the application is classified (OJ EPO 2014, A117). It should, however, be noted that the Notice from the EPO (OJ EPO 2007, 592) concerning business methods remains applicable. Therefore, the EPO as ISA will, in all cases where the subject matter of the international application involves technical means, consider the application and, to the extent possible, provide a search report for those parts of it which are more than mere business methods.

153
Q

An international application was filed on 27 February 2020 claiming the priority of a German national application filed on 27 February 2019. The EPO was selected as the ISA, and the ISR and accompanying WO-ISA were sent to the applicant with a mailing date of 30 March 2020.
For each of statements A to D, indicate whether the statement is true or false.

A. Claim amendments under Article 19 PCT reaching the IB on 29
May 2020 will be published together with the international application.
B. Claim amendments under Article 19 PCT reaching the IB on 29 June 2020 will be published together with the international application.
C. Claim amendments under Article 19 PCT reaching the IB on 17 July 2020 will be published together with the international application.
D. Claim amendments under Article 19 PCT reaching the IB on 17 August 2020 will be published together with the international application.

A

A. True.
B. True.
C. True.
D. False.
Rule 46.1 PCT lays down the time limit for making amendments to the claims under PCT Article 19 as 2 months from transmittal of the ISR, which is
30.03.2020 + 2 months  30.05.2020 (Saturday, Rule 80.5
PCT)  01.06.2020 (Monday),
or 16 months from the priority date, which is:
27.02.2019 + 16 months  27.06.2020 (Saturday, Rule 80.5
PCT)  29.06.2020 (Monday).

It follows that A and B are true. Rule 46.1 PCT additionally specifies that the claim amendments shall be considered to be received if received before technical preparations for publication are complete, which is 15 days prior to publication (AGIP, 9.014), publication usually being the first Thursday after 18 months from priority date, which is
27.02.2019 + 18 months  27.08.2020 (Thursday) - 15 days
 one day before: 11.08.2020. [Tuesday]

Calculation in detail:

  1. 02.2019 + 18 months → 27.08.2020 (Thursday) - 15 days →
  2. 08.2020 (Tuesday). Any change that should be taken into consideration for publication, should reach the IB at least one day before the 15-day time limit to allow the IB to process the change. The last day for receipt of such changes will therefore be on Tuesday, 11.08.2020 at midnight (Central European time (CET)). See AP-IP, 9.014 It follows that C is in time before the technical preparations are complete (true), and that D is after the technical preparations are complete, and therefore most likely too late. In practice, it would be a good idea to ask the IB when their technical preparations were going to be complete for the application in question, as publication schedules can sometimes shift. (The PCT requires publication to take place promptly after the expiration of 18 months from the priority date (Article 21(2)(a) PCT).)
154
Q

An international application was filed on 6 March 2020 claiming
priority from a German national application filed on 6 March 2019. The EPO was selected as the ISA, and the ISR and accompanying WOISA were sent to the applicant with a mailing date of3 April 2020. For each of statements A to D, indicate whether the statement is true or false.
A. The Russian Federation (RU) is competent as SISA for this
application.
B. The Japanese Patent Office (JP) is competent as SISA for this
application.
C. The European Patent Office (EP) is competent as SISA for this
application.
D. The Nordic Patent Institute (XN) is competent as SISA for this
application.

A

A. True.
B. False.
C. False.
D. True.
RU and XN have both made an agreement under Rule 45bis.9(a)
PCT to be a SISA (AG-IP, Annex D), so A and D are both true.
The EPO has also made an agreement under Rule 45bis.9(a) PCT to be a SISA, but it is not competent to be SISA in the above example because it also acted as ISA (Rule 45bis.9(b) PCT), so C is false. Finally, JP has not yet signed an agreement to become a SISA.

155
Q

How many IPEAs are there currently, and how many of them are

also ISAs?

A

There are currently (April 2020; AG-IP, Annex E) a total of 23
ISAs/IPEAs

156
Q

An international application was filed on 8 March 2019 claiming priority from a German national application filed on 8 March 2018. The EPO acted as the ISA, and the ISR and accompanying WO-ISA were sent to the applicant with a mailing date of 3 July 2019.
For each of statements A to D, indicate whether the statement is true or false.
A. A demand for international preliminary examination can be filed at the latest on 15 January 2020.
B. A demand for international preliminary examination can be filed at the latest on 4 October 2019.
C. A demand for international preliminary examination can be filed at the latest on 8 January 2020.
D. On 2 October 2019, the applicant files a demand for international
preliminary examination and pays the relevant fees. Claim amendments and accompanying arguments are only filed later, namely on 8 January 2020. The international preliminary
examination will start on 9 January 2020.The amended claims and the argumentation will be taken into account by the IPEA.

A

A. False.
B. False.
C. True.
D. False.
The demand must be filed three months from transmittal of the ISR or 22 months from the priority date, whichever expires later (Rule 54bis.1 PCT).
Three months from the transmittal of the ISR is:
03.07.2019 + 3 months  03.10.2019 (Thursday; National holiday; Rule 80.5 PCT)  04.10.2019 (Friday).
22 months from priority is
08.03.2018 + 22 months  08.01.2020 (Wednesday).
The longer of the two periods is 08.01.2020, which means that both A and B are false. C is true.
According to amended Rule 69.1(a) PCT (entered into force on July 1, 2019), the IPEA shall begin the international preliminary examination as soon as it is in possession of the demand, the fees and the ISR, without needing to wait until the time limit for filing a demand for international preliminary examination has expired. This ought to increase the time available for dialogue between the applicant and the examiner during international preliminary examination. It effectively reverses the previous default of waiting with the start of IPE until the
expiration of the time limit under Rule 54 bis.1(a) PCT (see previous Rule 69.1(a) PCT: “unless the applicant expressly requests an earlier start”) by starting IPE “unless the applicant expressly requests to postpone the start”.
Thus, with regard to the question at hand, the IPEA will start the international preliminary examination shortly after 2 October 2019, even though the amendments and arguments have not been received at that time. Therefore, the answer to question D is “false”. In order to make full use of the period for filing a demand for international preliminary examination, the applicant may expressly request postponement of IPE (by putting a respective mark in Form PCT/IPEA/401).

157
Q

An applicant files an international application claiming priority from a French application. He seeks protection in Belgium only. When should he enter the national/regional phase?
For each of statements (a) to (d), indicate whether the statement is true or false.
(a) The applicant should enter the national phase before the Belgian Patent Office within 30 months from the date of filing of the French application.
(b) The applicant should enter the regional phase before the EPO within 31 months from the date of filing of the French application.
(c) The applicant should first file a demand for international preliminary examination in order to be able to enter the national phase before the Belgian Patent Office within 31 months from the date of filing of the international application.
(d) The applicant should enter the regional phase before the EPO within 21 months from the date of filing of the international application.

A

False, True, False, False
(a) and (c) are false because Belgium has closed off the possibility of entering into the national phase before the national office concerned (Article 45(2) PCT; Euro-PCT Guide (12th edition, 2019) points 1.8.003 and 2.10.007). For Belgium, patent protection on the basis of an international application can only be obtained via the European route, i.e. by entering the European phase (Euro-PCT Guide, 12th edition, point 2.10.007). (d) is false since the term for entering the regional phase before the EPO is 31 months from the date of filing of the French application.

(b) is true because Belgium is an EPC contracting state and patent protection on the basis of an international application can only be obtained via the European route, i.e. by entering the European phase

158
Q

An applicant files an international application. No priority is claimed.
He seeks protection in Luxembourg only. When should he enter the
national/regional phase? For each of statements (a) to (d), indicate whether the statement is true or false.
(a) The applicant should enter the national phase before the Luxembourg Patent Office within 20 months from the date of filing of the international application.
(b) The applicant should first file a demand for international preliminary examination in order to be able to enter the national phase before the Luxembourg Patent Office within 30 months from the date of filing of the international application.
(c) The applicant should enter the regional phase before the EPO within 31 months from the date of filing of the international application.
(d) The applicant should enter the national phase before the Luxembourg Patent Office within 30 months from the date of filing of the international application.

A

True, True, True, False

(a) True. Luxembourg has a time limit for entering the national phase of 20 months from priority under PCT Chapter I (if no demand is filed).
(b) True. Luxembourg has a time limit for entering the national phase of 30 months from priority under PCT Chapter II (if a demand is filed).
(c) True. Luxembourg is an EPC contracting state and can also be covered through an EP regional phase application within the 31- month period.
(d) False. The 30-month time limit for national phase entry mentioned in (b) only applies if a demand has been filed. Otherwise, the national phase application in Luxembourg has to be filed within the 20-month time limit mentioned in (a).

159
Q

An applicant filed an international application in English on 11 July
2017, claiming a priority date of 13 July 2016. The ISA was the USPTO and the application contained 42 pages (including description, drawings, claims and abstract) and 22 claims.
For each of statements (a) to (d), indicate whether the statement is true or false.
(a) The applicant must pay the filing fee, including seven excess page fees, the search fee, the designation fee, the examination fee, seven excess claims fees and the third-year renewal fee, within the 31- month period. If all these fees are not paid within the 31-month period the application will be deemed withdrawn.
(b) The applicant must pay the filing fee, including seven excess page fees, the search fee, the designation fee and the examination fee, within the 31-month period. If all these fees are not paid within the 31-month period the application will be deemed withdrawn.
(c) The applicant should enter the regional phase before the EPO by 13 February 2019.
(d) The applicant should enter the regional phase before the EPO by 11 February 2020.

A

False, True, True, False

(a) False. Excess claims fees do not necessarily need to be paid within the 31-month period. The application will not be deemed withdrawn if excess claims fees are not paid within this period. The third-year renewal fee in this case will fall due on 31 July 2019, so these fees are also not due within the 31-month period.
(b) True. The filing fee (including excess page fee), designation fee, examination fee and supplementary search fee are required within the 31-month period and the application will be deemed withdrawn if these fees are not paid.
(c) True. The EP phase entry deadline is calculated from the priority date in cases where priority is claimed. The day 13.02.2019 (Wednesday) is the last day of the 31-month period of Art. 153 EPC.
(d) False. The EP phase entry deadline is calculated from the priority date, and not from the filing date, in cases where priority is claimed.

160
Q

The EPO acted as ISA for an international application. The ISA
found the claims to lack unity. Out of 30 claims, it searched claims 1 to 10. The EPO as ISA invited the applicant to pay two additional search fees. The applicant paid one additional search fee and had claims 11 to 19 searched. Claims 20 to 30 were not searched by the ISA.
For each of statements (a) to (d), indicate whether the statement is true or false.
(a) The applicant will be provided with an opportunity to have claims 20 to 30 searched during the EP regional phase.
(b) If claims 20 to 30 are not searched during the EP regional phase, the applicant will be invited to limit the application to the invention of claims 1 to 10 or the invention of claims 11 to 19 during the EP regional phase.
(c) If claims 20 to 30 are not searched during the EP regional phase,
the applicant can choose to limit the Euro-PCT application to the invention covered by claims 1 to 10 and to file a divisional application for the invention covered in claims 11 to 19.
(d) If claims 20 to 30 are not searched during the EP regional phase, the applicant can choose to limit the Euro-PCT application to the invention covered by claims 11 to 19 and to file a divisional application for the invention covered in claims 20 to 30.

A

True, True, True, True

(a) True. The Examining Division upon entry of the international application into the regional phase before the EPO will find out that the application on which the European grant procedure is to be based does not meet the requirement of unity of invention and will invite the applicant to pay a further search fee for the invention in claims 20 to 30 (Rule 164(2)(a) EPC). If the applicant pays the further search fee, the Examining Division will issue a search report for claims 20 to 30. The applicant can choose one of these three inventions to pursue in the regional phase before the EPO (Rule 164(2)(c) EPC). If desired, the applicant can file a divisional application for the two other inventions.
(b) True. In the international phase, the applicant paid an additional search fee for claims 11 to 19. Therefore, the EPO as ISA searched claims 1 to 10 and claims 11 to 19. The applicant can choose which of these two inventions to pursue in the regional phase before the EPO (Rule 164(2)(c) EPC).
(c) True. Claims 1 to 10 and claims 11 to 19 have been searched by the EPO as ISA in the international phase. Following Rule 164(2)(c) EPC, the applicant will be invited to pursue either claims 1 to 10 or claims 11 to 19 in the Euro-PCT application. If the applicant chooses to pursue claims 1 to 10 in the Euro-PCT application, he can file a divisional application in respect of claims 11 to 29 (Article 76 EPC). The search fee paid upon filing the divisional application will, probably, be reimbursed by the EPO (Rules Relating to Fees, Article 9(1)).
(d) True. In the international phase, claims 1 to 10 and claims 11 to 19 have been searched by the EPO as ISA. Following Rule 164(2)(c) EPC, the applicant will be invited to pursue either claims 1 to 10 or claims 11 to 19 in the Euro-PCT application. If the applicant chooses to pursue claims 11 to 19 in the Euro-PCT application, he can file a divisional application in respect of claims 20 to 30 (Article 76 EPC). Upon filing the divisional application, all the fees have to be paid accordingly. There will be no refund of the search fee because claims 20 to 30 have neither been searched by the EPO in the international phase nor in the regional phase.

161
Q

An applicant filed an international application in English on 12 July
2019, claiming a priority date of 12 July 2018. The 31-month period for EP phase entry ends on 12 February 2021. Today is 16 July 2020 and the applicant wants to accelerate the procedure before the EPO.
For each of statements (a) to (d), indicate whether the statement is true or false.
(a) If the applicant completes all the EP phase entry requirements today and requests PACE, the EPO will deal with the application
straightaway.
(b) If the applicant completes all the EP phase entry requirements today, explicitly requests early national phase processing and requests PACE, the EPO will deal with the application straightaway.
(c) If the applicant completes all the EP phase entry requirements today, explicitly requests early national phase processing and requests PACE, the EPO will deal with the application straightaway.
Moreover, if he waives the Rule 161 EPC communication, future procedure will be further accelerated.
(d) The EPO will not process Euro-PCT applications before the 31 month period unless explicitly requested to do so by the applicant (Article 23(2)/Article 40(2) PCT).

A

False, True, True, True
(a) False. Requesting PACE and performing the EP phase entry
requirements before the end of the international phase (31 months from priority) are not sufficient to accelerate the EPO proceedings. If the applicant wishes to enter into the regional/national phase early and to request acceleration of proceedings, then he must file an express request for early national phase entry under Article 23(2)/Article 40(2) PCT (Euro-PCT Guide (12th Edition, 2019), item 5.1.24), along with a PACE request.
(b) True. The actions specified will be sufficient for the EPO to start processing the application, even though the international phase has not ended.
(c) True, for the same reasons as (b) above. Waiving the Rule 161/162 EPC communication further accelerates proceedings, as the applicant does not have to wait for this communication to be issued and then for the specified six-month response period to expire before further action on the application by the EPO.
(d) True. As specified in (a), the regional/national offices will not process PCT applications before the expiry of the international phase, unless explicitly requested to do so by the applicant (Article 23(2)/Article 40(2) PCT).

162
Q

Could a patent be obtained for a method involving the treatment of a human by administration of a chemical product that could be used for both medical and cosmetic purposes, if the claim states that the method is directed to administering the compound to obtain a cosmetically beneficial effect?

A

Yes. See GL G-II, 4.2.1 and T 144/83

163
Q

Would the following claim be excluded under Article 53(c) EPC?
A method of treating blood including the following steps:
▪ connecting a blood bag containing blood to a blood centrifuge,
▪ separating red and white blood cells by means of said centrifuge,
▪ introducing the treated blood into a clean blood bag, and
▪ storing the blood bag in a blood bank.

A

No. To be excluded from patentability, a treatment or diagnostic method must actually be carried out on the living human or animal body. The treatment of body tissues or fluids after they have been removed from the human or animal body, or diagnostic methods applied thereon, is not excluded from patentability insofar as these tissues or fluids are not returned to the same body (GL G-II, 4.2.1).

164
Q

Which of the following are excluded from patentability under Article
53(c) EPC?
A. A method of MRI imaging of a patient involving the injection of a
contrast agent directly into the heart, the method not involving a step of finding symptoms or making a diagnosis.
B. A method of MRI imaging of a patient involving the inhalation of a contrast agent, the method not involving a step of finding symptoms or making a diagnosis.
C. The treatment of blood by dialysis, the blood being returned to the same body.
D. The treatment of a sheep in order to promote growth.

A

A. Yes. The method is excluded because it involves an invasive step (injection of a radioactive marker directly into the heart) which requires professional medical expertise to be carried out and entails a substantial health risk even when carried out with the required expertise ((GL G-II, 4.2.1.1; G 1/07).
B. No. In the case underlying G 1/07, the claim contained a step of delivery of a substance to a patient, which could be achieved by injection or inhalation. The communication from the examining division stated that “if delivery took place by injection this would be a surgical step”, which implies that delivery by inhalation would not be considered a surgical step. The nature of the substance delivered to the patient was not important, only the way of delivering it.
C. Yes. The method is also excluded because dialysis removes
blood from and then returns it to the patient, which is a therapeutic treatment as defined in Article 53 (c) EPC, and the process of dialysis involves a surgical intervention which requires professional medical expertise to be carried out and entails a substantial health risk even when carried out with the required expertise (GL G-II, 4.2.1; G 1/07).
D. No. Promoting the growth of sheep is not considered to be therapeutic in nature, but rather an agricultural activity, since it does not set out to maintain the life or health of the animal (GL G-II, 4.2.1; T 383/03).

165
Q

Which of the following can a patent be obtained for?
A. The use of an antibacterial substance for the treatment of acne.
B. A specified chemical compound for use in treatment of a bacterial infection.
C. A method involving the use of one or more sensors placed at one or more points along the internal respiratory flow path for internal registration and measurement of the respiratory tidal volume.
D. A method involving the use of an antibiotic for the “non- therapeutic improvement of the performance of an animal” (the performance being, for example, the amount of milk produced by a cow).

A

A. No. This may be merely a cosmetic treatment, but it may also incidentally involve a medical treatment. As long as the claim encompasses the therapeutic use it is excluded under Article 53(c) EPC. A limitation to “Use of the substance as a cosmetic product” would be sufficient to avoid exclusion under Article 53(c) EPC (T0036/83).

B. Yes. This relates to the chemical compound for use in a method of treatment, and not to a method of treatment per se. It is therefore not excluded under Article 53(c) EPC.

C. No. This method claim is excluded under Article 53(c) EPC because it inherently requires the invasive step of inserting sensors inside a patient’s body. Even though this step does not involve penetration of the patient’s tissue, it is considered to be surgical, just as is the case for catheterisation of a patient (T 0005/04).

D. Yes. The antibiotic in question has two distinct uses. One is medical and relates to improving the animal’s health. The other relates only to increasing the amount of milk, without any effect on the animal’s health. Hence, limiting the claimed invention to the “non-therapeutic” use is sufficient to avoid exclusion under Article 53(c) EPC (T 774/89).

166
Q

A method of diagnostic imaging an object using imaging equipment, said method comprising: disposing the object on a movable object handling system; docking the object handling system; producing diagnostic images of said object with the imaging equipment; undocking the object handling system; entering object identification data into a data storage means disposed on the object handling system prior to producing diagnostic images of said object; connecting said data storage means to said one of the first imaging equipment and second imaging equipment prior to producing diagnostic images therewith; and associating said data with the images of said object produced by said imaging equipment.
A. Does the claim relate to a diagnostic method excluded under Article 53(c) EPC?
B. Does the claim relate to a surgical method excluded under article 53(c) EPC because it may involve the use of X-rays, which is an invasive step?

A

A. No. Since the method involves taking diagnostic images, we need to consider whether it is a diagnostic method excluded under Article 53(c) EPC. According to G 1/04, to be a diagnostic method claim, the method must also include (i) the diagnosis for curative purposes stricto sensu representing the deductive medical or veterinary decision phase as a purely intellectual exercise, and (ii) the preceding steps which are constitutive for making that diagnosis, and that are performed on a human body. The present claimed method, however, does not qualify for patent exclusion as a diagnostic method because it only provides intermediate results (images), lacking both the comparison of the images with a standard, and the finding of any significant deviation (or symptom) during the comparison, such as to allow a diagnosis to be made.

B. No. The method does not include or require any surgical step, since the step of imaging does not involve physical intervention of the body. Furthermore, it is not suitable for maintaining or restoring the health, physical integrity or physical well-being of a human being or animal, or to prevent diseases.

167
Q

Could a patent be obtained for the following claim?
“A method of determining ear temperature, comprising the steps of: i. providing a radiation detector comprising an extension for passing infrared radiation from an external target to a sensor and electronics for converting the radiation sensed by the sensor to temperature;
ii. inserting the extension into an ear; and
iii. detecting the radiation; characterised in that the method further comprises the step of pivoting or rotating the extension to scan an ear canal, the sensor sensing radiation during scanning, and determining the peak radiation sensed by the sensor and converting the peak radiation sensed to a peak ear temperature.”

A

Yes. The claim relates to a method of data collection, without any steps of finding a significant deviation and attributing this deviation to a particular clinical picture. In other words, since the findings are not used to obtain a diagnosis, the claim is not a diagnostic method excluded under Article 53(c) EPC (T 1255/06).

168
Q

Could a patent be obtained for the following claim?
“A method of assessing the presence of glaucomatous damage to the visual system of a subject, the method comprising steps of:
i. generating and displaying a pattern having characteristics consistent with observing a spatial frequency-doubled illusion obtained by a contrast-modulated grating pattern;
ii. presenting said pattern within the visual field of view of a subject; iii. measuring a contrast threshold value at which said subject can just discern each pattern while fixating on a fixation spot wherein said fixation spot is located at 0º azimuth and elevation in said visual field; and
iv. comparing the threshold value with a standard value based on subjects of normal vision.”

A

No. This claim includes all the features of a diagnostic method
practised on the human or animal body as defined in G 1/04. Since the claim is directed to “A method for assessing the presence of glaucomatous damage” it includes the phase of attributing the findings to a particular clinical picture. Steps (i) to (iii) comprise the technical steps need for making a diagnosis, while step (iv) represents a comparison of the collected thresholds with standard values exhibited by subjects of normal vision and, implicitly, the finding of any significant deviation resulting from the comparison, i.e. the symptom (T 1197/02).

169
Q

The prior art describes the structure of the compound atrazine, as
well as methods for its manufacture. Atrazine is the only member of
the triazines that has ever been synthesised or isolated. A patent
application describes new members of the triazine family and shows that members of the triazine family can be used as herbicides. The application has four claims, claims 1 to 4, reading as follows:
1. A herbicidal composition comprising a triazine compound selected from the group consisting of amitrol, cyanazine, simazine and trietazine.
2. A dry granulate comprising the herbicidal composition of claim 1.
3. A method of destroying non-crop plants comprising the step of spraying a herbicidal composition comprising a triazine.
4. Use of a triazine compound selected from the group consisting of amitrol, cyanazine, simazine and trietazine as a herbicide.

A. Is claim 1 novel in view of the prior art?
B. Do any of the claims contain a Markush group?
C. Do the claims meet the requirements of unity of invention?
D. If the applicant’s test data shows that simazine does not have
herbicidal properties, will this be important for determining the patentability of claims 1-4?

A

A. Yes. The claim is directed to a list of novel triazine compounds, the selected members being shown to have a new use as a herbicide which was not previously known from the prior art.
B. Yes. Claims 1, 2 and 4 provide a list of herbicidal compounds that share a common structure (all triazines) and a common property (herbicidal activity).
C. Yes. The special technical feature common to the claims is the group of selected triazine compounds with herbicidal properties. This special technical feature is both novel and inventive in view of the newly discovered herbicidal properties of the triazine family, and defines a contribution which each of the claims make over the art.
D. Yes. In order to meet the requirements of inventive step, all embodiments encompassed by the claims must solve the technical problem addressed by the invention (T 939/92).

170
Q

If a sequence listing is not filed by the filing date for a European patent application, will the application be refused? If not, what action will the EPO take?

A

No. If the sequence listing is not filed with the application or if there is any deficiency in the sequence listing, the Receiving Section will invite the applicant to file the sequence listing or to remedy the deficiencies and pay a late furnishing fee within a non-extendable period of two months from notification of the invitation (Rule 30(3) EPC). If the two-month period is not observed, the application will be refused in accordance with Rule 30(3) EPC. Applicants can request further processing under Article 121 EPC.

171
Q

For a patent application relating to an invention involving biological material which is not available to the public and which cannot be sufficiently described in the application, is it possible to deposit the biological material at any point within 16 months of the date of filing of the European patent application or the date of priority?

A

No. The required deposit should be made no later than the date of filing of the application (Rule 31(1)(a) EPC). If priority is claimed, the required deposit should be made no later than the priority date in order for the priority claim to be valid (OJ EPO 2010, 498).

The applicant is required to provide details regarding the deposit in accordance with Rule 31(1)(c) and (d) EPC. If such information is not contained in the application as filed, it should be submitted as provided for in Rule 31(2) EPC.

172
Q

A claim is directed to a computer program adapted to co-ordinate and control the internal communication between program and data files in different processors of a data-processing system having a plurality of interconnected data processors in a telecommunications network. Is the subject-matter excluded from patentability?

A

No. In view of G3/08, the presence of data processors imparts a technical character (T6/83; GL G-II, 2; GL G-II, 3.6).

173
Q

An applicant is seeking to justify an undisclosed disclaimer to prior art disclosing an embodiment of the claimed invention by arguing that the anticipation was accidental. The prior art in question was actually mentioned in the discussion of the prior art in the application in suit. Will the disclaimer be allowed?

A

No. The applicant is clearly aware of the prior art, because they actually mention it in their patent application. The anticipation could not possibly be accidental (T 500/00; CLB II.E. 1.5.3 a) Accidental anticipation).

174
Q

A European patent application contains a claim to a composition for applying to the skin to create a fake tan. It also contains a claim to a method of applying the composition to the skin by spraying. While not disclosed in the application, the composition is known to also have a therapeutic effect and could be used to treat certain skin diseases. Can the method having a therapeutic effect be disclaimed?

A

Yes. Since the method having a therapeutic effect is nonpatentable by virtue of Article 53 EPC, it can be disclaimed in accordance with G 1/03, headnote 2.1, in that undisclosed subject matter can be disclaimed which, under Articles 52 to
57 EPC, is excluded from patentability for non-technical reasons.

175
Q

In order to distinguish a claim from prior art cited under Article 54(3) EPC, the applicant has submitted an undisclosed disclaimer which is considerably broader than the cited prior art. Will the disclaimer be allowed?

A

No. Even though the disclaimer restored novelty over the cited prior art, the broad disclaimer also disclaimed subject-matter which was not anticipated by the prior art and thus did not comply with point 2.2 of the order in G 1/03 (T 8/07).

176
Q

During examination of a patent application, document D1 was cited under Article 54(3) EPC and document D2 under Article 54(2) EPC. Both documents relate closely to the technical area of the patent application. The applicant has proposed an undisclosed disclaimer which restores novelty against D1 and at the same time also restores novelty against D2. Will the disclaimer be allowed?

A

No. The disclaimer is relevant to the assessment of inventive step relative to D2 and therefore adds subject-matter contrary to Article 123(2) EPC in accordance with point 2.3 of the order in G 1/03 (T 788/05).

177
Q

An applicant submits a disclaimer which makes the scope of the claim impossible to determine. Will the disclaimer be allowed?

A

No. The claim lacks clarity and therefore does not comply with
point 2.4 of the order in G 1/03.

178
Q

An applicant amends a treatment time of 1 to 10 minutes in a claim to a treatment time of 1 to 6 minutes, effectively disclaiming the sub-range of 6 to 10 minutes which overlaps with the prior art. The application as filed does not contain any indication of the sub-range 1 to 6 minutes. Will the disclaimer be allowed?

A

No. It is impossible for the skilled reader to regard the sub-range of 1 to 6 minutes as having been disclosed, notwithstanding the fact that the sub-range actually falls within the range originally disclosed. Thus the disclaimer contravenes Article 123(2) EPC and is not allowable